Chapter 13 - Lungs

Réussis tes devoirs et examens dès maintenant avec Quizwiz!

OBSTRUCTIVE LUNG (AIRWAY) DISEASES: Chronic Bronchitis Most important causes

(1) Cigarette smoking Other reasons: - Air pollutants, such as sulfur dioxide and nitrogen dioxide

ATELECTASIS (COLLAPSE) • Resorption atelectasis: - Occurs when - Most common cause

- Occurs when an obstruction prevents air from reaching distal airways. - The most common cause of resorption collapse is obstruction of a bronchus.

OBSTRUCTIVE LUNG (AIRWAY) DISEASES: Emphysema: - Pulmonary function tests

- Pulmonary function tests reveal : (1) reduced FEV1 (2) normal or near-normal FVC. Hence, the FEV1 to FVC ratio is reduced

ATELECTASIS (COLLAPSE) Basal atelectasis Etiology Common in

- Resulting from a failure to breath deeply - Commonly occurs in: bedridden patients, in patients with ascites, and during and after surgery.

OBSTRUCTIVE LUNG (AIRWAY) DISEASES: Chronic Bronchitis Sequences in smokers:

The transcription of the mucin gene in bronchial epithelium and the production of neutrophil elastase are increased as a consequence of exposure to tobacco smoke.

OBSTRUCTIVE LUNG (AIRWAY) DISEASES: Bronchiectasis The usual organisms cause Bronchiectasis include

The usual organisms include staphylococci, streptococci, pneumococci, enteric organisms, anaerobic and microaerophilic bacteria, and (particularly in children) Haemophilus influenzae and Pseudomonas aeruginosa.

OBSTRUCTIVE LUNG (AIRWAY) DISEASES: Asthma Associated Gene defect (1)

IL-4 receptor - involved in asthma pathogenesis.

RESTRICTIVE PULMONARY DISEASES: Causes of restrictive lung disease The mnemonic "PAINT"

The mnemonic "PAINT" has been used to divide the causes of restrictive lung disease into pleural, alveolar, interstitial, neuromuscular, and thoracic cage abnormalities

OBSTRUCTIVE LUNG (AIRWAY) DISEASES: Emphysema: Panacinar Morphology

Typical panacinar emphysema produces pale, voluminous lungs that often obscure (מסתירות) the heart when the anterior chest wall is removed at autopsy.

Diffuse Alveolar Hemorrhage Syndromes: - Conditions refers to "Pulmonary Hemorrhage Syndromes" (3)

(1) Goodpasture syndrome (2) Idiopathic pulmonary hemosiderosis (3) Granulomatosis with polyangiitis

ACUTE RESPIRATORY DISTRESS SYNDROME (ARDS) Etiology

ARDS may occur in a multitude of clinical settings and is associated with (1) primary pulmonary diseases (2) severe systemic inflammatory disorders such as sepsis.

OBSTRUCTIVE LUNG (AIRWAY) DISEASES: Asthma Status asthmaticus

Occasionally a severe paroxysm (הֶתקֵף פִּתאוֹמִי) occurs that does not respond to therapy and persists for days and even weeks The associated hypercapnia, acidosis, and severe hypoxia may be fatal, although in most cases the condition is more disabling than lethal.

ATELECTASIS (COLLAPSE) Results in

Shunting of inadequately oxygenated blood from pulmonary arteries into veins, thus giving rise to a ventilation-perfusion imbalance and hypoxia.

Pathophysiologic Consequences of Pulmonary Thromboembolism: Effects, Hypoxemia Mechanisms, and Infarction (4)

1. Consequences of pulmonary arterial occlusion include increased pulmonary artery pressure and ischemia of downstream pulmonary parenchyma. 2. Occlusion of a major vessel can lead to abrupt increase in pulmonary artery pressure, diminished cardiac output, right-sided heart failure (acute cor pulmonale), and sometimes sudden death. 3. Hypoxemia develops as a result of multiple mechanisms: a. Perfusion of atelectatic lung zones - Alveolar collapse occurs in ischemic areas because of a reduction in surfactant production and because pain associated with embolism leads to reduced movement of the chest wall. b. The decrease in cardiac output causes a widening of the difference in arterial-venous oxygen saturation. c. Right-to-left shunting of blood through a patent foramen ovale 4. Ischemic necrosis (infarction) of bronchial arteries occurs in as few as 10% of patients with thromboemboli, and only if there is a compromise in cardiac function, bronchial circulation, or underlying pulmonary disease causing underventilation of the lung region at risk.

OBSTRUCTIVE LUNG (AIRWAY) DISEASES: Chronic Bronchitis Clinical features

1. Cough and sputum production persist indefinitely without ventilatory dysfunction or 2. COPD with significant outflow obstruction marked by hypercapnia (an elevation in the arterial carbon dioxide tension), hypoxemia, and cyanosis - Patients with chronic bronchitis and COPD have frequent exacerbations, more rapid disease progression, and poorer outcomes than those with emphysema alone - Progressive disease is marked by the development of pulmonary hypertension, sometimes leading to cardiac failure; recurrent infections; and ultimately respiratory failure.

Smoking-Related Interstitial Diseases: Desquamative Interstitial Pneumonia (DIP) and Respiratory Bronchiolitis (2)

1. Desquamative interstitial pneumonia (DIP): - Accumulation of large numbers of macrophages containing dusty-brown pigment (smokers macrophages) in the air spaces. - Alveolar septa are thickened by a sparse inflammatory infiltrate (usually lymphocytes); interstitial fibrosis, when present, is mild. - Patients have a good prognosis and an excellent response to steroids and smoking cessation; however, some patients progress despite therapy. 2. Respiratory bronchiolitis: - Common lesion found in smokers, characterized by the presence of pigmented intraluminal macrophages in a "bronchiolocentric" distribution (first- and second-order respiratory bronchioles). - Mild peribronchiolar fibrosis also is seen. - Affected patients present with gradual onset of dyspnea and dry cough, and the symptoms recede with smoking cessation.

Morphology of Asbestosis: Asbestos Bodies, Fibrosis Distribution, and Pleural Manifestations (6)

1. Diffuse pulmonary interstitial fibrosis 2. Characterized by the presence of asbestos bodies, which are golden brown, fusiform or beaded rods with a translucent center. 3. Asbestos bodies consist of asbestos fibers coated with an iron-containing proteinaceous material, derived from phagocyte ferritin. 4. Fibrosis distribution begins in the lower lobes and subpleurally, spreading to middle and upper lobes as fibrosis progresses. - Contraction of fibrous tissue distorts normal architecture, creating enlarged air spaces enclosed within thick fibrous walls, eventually becoming honeycombed. 5. Fibrosis in the visceral pleura causes adhesions between the lungs and the chest wall, leading to pulmonary hypertension and cor pulmonale. 6. Pleural plaques are the most common manifestation of asbestos exposure. - Well-circumscribed plaques of dense collagen, often containing calcium.

Pleural Lesions: Diseases of the Pleura (2)

1. Disease of the pleura usually is a complication of an underlying pulmonary disease. 2. Secondary infections and pleural adhesions are common findings at autopsy. 3. Important Primary disorders are (1) primary intrapleural bacterial infections (2) malignant mesothelioma, a primary neoplasm of the pleura.

Drug- and Radiation-Induced Pulmonary Disease: Causes, Symptoms, and Treatment (4)

1. Drugs like bleomycin (an anti-cancer agent) and amiodarone (an anti-arrhythmic agent) can cause pneumonitis and interstitial fibrosis. 2. Radiation pneumonitis is a complication of irradiation therapy for thoracic tumors. 3. Acute radiation pneumonitis occurs in up to 20% of patients, typically 1 to 6 months after therapy, and presents with fever, dyspnea, pleural effusion, and pulmonary infiltrates. 4. Acute radiation pneumonitis may resolve with corticosteroid therapy or progress to chronic radiation pneumonitis, which is associated with pulmonary fibrosis.

Silicosis: Morphology, Microscopic Features, and Progression (6)

1. Early-stage silicotic nodules are tiny, barely palpable, discrete, and pale-to-black, predominantly in the upper lung zones. 2. Microscopically, nodules show concentric hyalinized collagen fibers surrounding an amorphous center. 3. The "whorled" appearance of the collagen fibers is distinctive for silicosis. 4. Polarized microscopy reveals weakly birefringent silica particles in the center of the nodules. 5. As the disease progresses, nodules may coalesce into hard, collagenous scars, leading to PMF. 6. Fibrotic lesions may also occur in hilar lymph nodes and the pleura.

Haemophilus influenzae: Types, Pneumonia in Children, and Adults at Risk (3)

1. Encapsulated and unencapsulated forms: - Both can cause community-acquired pneumonias 2. Life-threatening pneumonia in children: - Encapsulated H. influenzae can cause severe pneumonia, often after a respiratory viral infection 3. Adults at risk for H. influenzae infections: - Chronic pulmonary diseases (chronic bronchitis, cystic fibrosis, bronchiectasis) - Most common bacterial cause of acute exacerbations of COPD - Vaccination against encapsulated H. influenzae type b significantly reduced the risk of epiglottitis and suppurative meningitis in children

Chronic Pneumonias: Characteristics and Importance of Tuberculosis (2)

1. Feature: Localized lesion in immunocompetent individuals Granulomatous inflammation Regional lymph node involvement. 2. Tuberculosis: The most important entity within the spectrum of chronic pneumonias

Bronchopneumonia Morphology: Consolidated Areas, Distribution, Gross Appearance, and Histology (4)

1. Foci of bronchopneumonia are consolidated areas of acute suppurative inflammation. 2. The consolidation may be confined to one lobe but is more often multilobar and frequently bilateral and basal because of the tendency of secretions to gravitate to the lower lobes. 3. Well-developed lesions are slightly elevated, dry, granular, gray-red to yellow, and poorly delimited at their margins. 4. Histologically, a neutrophil-rich exudate fills the bronchi, bronchioles, and adjacent alveolar spaces

Carcinoma of the Larynx: Prognosis (4)

1. Glottic tumors (on the vocal cords) have a better prognosis as they develop symptoms early and have a sparse lymphatic supply, leading to less spread beyond the larynx. 2. Supraglottic tumors (above the cords) have a higher risk of metastasis to regional lymph nodes. 3. Subglottic tumors (below the cords) tend to present as advanced disease. 4. Treatment with surgery, radiation therapy, or a combination can lead to cure in many patients, but about one-third of patients die of the disease due to widespread metastases and cachexia, sometimes complicated by pulmonary infection.

Idiopathic Pulmonary Fibrosis (IPF): Clinical Features, Diagnosis, Treatment, and Prognosis Idiopathic Pulmonary Fibrosis (IPF)

1. Gradual onset of nonproductive cough and progressive dyspnea 2. Characteristic "dry" or "Velcro-like" crackles during inspiration on physical examination 3. Cyanosis, cor pulmonale, and peripheral edema may develop in later stages of the disease 4. Characteristic clinical and radiologic findings, such as subpleural and basilar fibrosis, reticular abnormalities, and "honeycombing" are needed for diagnosis 5. Lung transplantation is the only definitive treatment 6. Overall prognosis remains poor, with survival typically ranging from 3 to 5 years

Morphology of Idiopathic Pulmonary Fibrosis (IPF): Gross Features, Histology, Fibrosis, and Inflammation (7)

1. Grossly, pleural surfaces of the lung appear cobblestoned due to retraction of scars along the interlobular septa. 2. Cut surface shows firm, rubbery white areas of fibrosis, preferentially within the lower lobe, subpleural regions, and along the interlobular septa. 3. Histologically, patchy interstitial fibrosis with varying intensity is the hallmark of IPF. 4. Early lesions demonstrate exuberant fibroblastic proliferation (fibroblastic foci). 5. Over time, areas become more collagenous and less cellular, with both early and late lesions present. 6. Dense fibrosis causes collapse of alveolar walls and formation of cystic spaces lined by hyperplastic type II pneumocytes or bronchiolar epithelium (honeycomb fibrosis). 7. Interstitial inflammation is usually patchy, with alveolar septal infiltrate of mostly lymphocytes, occasional plasma cells, mast cells, and eosinophils; secondary pulmonary hypertensive changes often present.

Morphology of Yeast Forms in Histoplasmosis, Coccidioidomycosis, and Blastomycosis (3)

1. H. capsulatum: Round to oval, small yeast forms measuring 2 to 5 μm in diameter. 2. C. immitis: Thick-walled, nonbudding spherules, 20 to 60 μm in diameter, often filled with small endospores. 3. B. dermatitidis: Round to oval and larger yeast forms than Histoplasma (5 to 25 μm in diameter) that reproduce by characteristic broad-based budding.

Morphological Findings in Goodpasture Syndrome: Lung Examination and Immunoglobulin Deposition (7)

1. Heavy lungs with red-brown consolidation due to diffuse alveolar hemorrhage 2. Focal necrosis of alveolar walls 3. Intra-alveolar hemorrhage 4. Fibrous thickening of septa 5. Hypertrophic type II pneumocytes 6. Abundant hemosiderin due to earlier episodes of hemorrhage 7. Immunoglobulin deposition (usually IgG, sometimes IgA or IgM) along alveolar septa

Chronic Interstitial Lung Diseases: Characteristics, Histology, and Clinical Consequences (4)

1. Heterogeneous group of disorders characterized by bilateral, often patchy, pulmonary fibrosis mainly affecting alveoli walls. 2. Many entities are of unknown cause and pathogenesis; some have intraalveolar and interstitial components. 3. Hallmark is reduced compliance (stiff lungs), leading to dyspnea. 4. Damage to alveolar epithelium and interstitial vasculature causes abnormalities in ventilation-perfusion ratio, leading to hypoxia, respiratory failure, pulmonary hypertension, and cor pulmonale. - Advanced cases result in diffuse scarring and gross destruction of the lung, referred to as end-stage or "honeycomb" lung.

Histoplasmosis, Coccidioidomycosis, and Blastomycosis: Dimorphic Fungi Infections, Epidemiology, and Geographic Distribution (3)

1. Histoplasma capsulatum, Coccidioides immitis, and Blastomyces dermatitidis are dimorphic fungi causing infections with either isolated pulmonary involvement or disseminated disease in immunocompromised individuals. 2. T cell-mediated immune responses are critical for containing the infection; individuals with compromised cell-mediated immunity, such as those with HIV, are prone to systemic disease. 3. Geographic distribution: a. H. capsulatum: endemic in Ohio and central Mississippi River valleys, and southeastern United States along the Appalachian Mountains; grows in warm, moist soil with bat and bird droppings. b. C. immitis: endemic in southwestern and far western United States, especially California's San Joaquin Valley; known as "valley fever." c. B. dermatitidis: overlaps with histoplasmosis distribution in the United States.

Hypersensitivity Pneumonitis: Definition, Immunology, Clinical Features (3)

1. Hypersensitivity pneumonitis is an immunologically mediated inflammatory lung disease that primarily affects the alveoli and is often called allergic alveolitis. 2. The disease is most often an occupational disease that results from heightened sensitivity to inhaled antigens such as those found in moldy hay. 3. Unlike bronchial asthma, the damage in hypersensitivity pneumonitis occurs at the level of alveoli, manifesting predominantly as a restrictive lung disease with typical decreases in diffusion capacity, lung compliance, and total lung volume.

Immunity and Hypersensitivity in Tubercular Infection (Summary of Pathogenesis): TH1 Cells, Mediators, and Implications (5)

1. Immunity to tubercular infection is primarily mediated by TH1 cells, which stimulate macrophages to kill mycobacteria. 2. This immune response is largely effective but comes with the cost of hypersensitivity and tissue destruction. 3. Defects in any step of the TH1 T cell response (IL-12, IFN-γ, TNF, or nitric oxide production) lead to poorly formed granulomas, absence of resistance, and disease progression. 4. Individuals with inherited mutations in any component of the TH1 pathway are extremely susceptible to infections with mycobacteria. 5. Loss of hypersensitivity (indicated by tuberculin negativity in an M. tuberculosis-infected patient) is an ominous sign of fading resistance to the organism.

Cytomegalovirus Infection in Immunosuppressed Individuals: Commonly Affected Organs, Manifestations, and Diagnosis (6)

1. Immunosuppression-related CMV infection occurs in transplant recipients and patients with AIDS. 2. Can be either primary infection or reactivation of a latent infection. 3. CMV is the most common opportunistic viral pathogen in AIDS. 4. Serious disseminated CMV infections primarily affect lungs (pneumonitis), gastrointestinal tract (colitis), and retina (retinitis); the central nervous system is usually spared. 5. Diagnosis of CMV infection is made by demonstrating viral inclusions in tissue sections, successful viral culture, rising anti-viral antibody titer, and qualitative or quantitative PCR assay-based detection of CMV DNA.

Cytomegalovirus Mononucleosis: Clinical Manifestations, Prevalence, and Latency in Immunocompetent Hosts (5)

1. In healthy young children and adults, CMV infection is usually asymptomatic. 2. 50% to 100% of adults worldwide demonstrate anti-CMV antibodies in the serum, indicating previous exposure. 3. The most common clinical manifestation in immunocompetent hosts beyond the neonatal period is an infectious mononucleosis-like illness with fever, atypical lymphocytosis, lymphadenopathy, and hepatomegaly with mild hepatitis. 4. Most patients recover without any sequelae, but virus excretion can occur in body fluids for months to years. 5. Infected individuals become seropositive for life, with the virus remaining latent within leukocytes, which serve as the major reservoirs.

Influenza Infections: Virus Types, Antigenic Shift & Drift, Pandemics, and Concerns (5)

1. Influenza virus: single-stranded RNA virus with types A, B, or C, determined by the nucleoprotein. 2. Viral surface: lipid bilayer containing hemagglutinin and neuraminidase, determining the subtype (e.g., H1N1, H3N2). 3. Host antibodies: prevent or ameliorate future infections, type A viruses infect various animals and cause pandemic and epidemic influenza infections. 4. Antigenic drift and shift: mutations of hemagglutinin and neuraminidase allow escape from host antibodies; pandemics occur when both are replaced through recombination with animal viruses. 5. Concerns about future pandemics: avian influenza strain H5N1 has infected humans with limited transmission, but a recombination with a highly infectious human strain could cause a new pandemic.

Morphology and Spread of Tuberculosis: Initial Lesion, Progressive Pulmonary TB, Miliary Pulmonary Disease (5)

1. Initial lesion: - Small focus of consolidation, <2 cm in diameter, within 1-2 cm of apical pleura - Sharply circumscribed, firm, gray-white to yellow areas with variable central caseation and peripheral fibrosis - Active lesions show coalescent tubercles with central caseation, tubercle bacilli demonstrated in early exudative and caseous phases 2. Progressive pulmonary tuberculosis: - Erosion into a bronchus evacuates the caseous center, creating a ragged, irregular cavity lined by caseous material that is poorly walled off by fibrous tissue - Healing by fibrosis may distort pulmonary architecture - Infection may spread through direct extension, airways, lymphatics, and vascular system 3. Miliary pulmonary disease: - Occurs when organisms reach bloodstream through lymphatic vessels and recirculate to the lung via pulmonary arteries - Small (2-mm) foci of yellow-white consolidation scattered throughout the lung parenchyma - With progressive pulmonary tuberculosis, the pleural cavity is invariably involved and serous pleural effusions, tuberculous empyema, or obliterative fibrous pleuritis may develop. 4. Endobronchial, endotracheal, and laryngeal tuberculosis may develop when infective material is spread either through lymphatic channels or from expectorated infectious material.

Pathogenesis of Idiopathic Pulmonary Fibrosis (IPF): Repeated Injury, Genetic Factors, and Profibrotic Factors (6)

1. Interstitial fibrosis in IPF believed to result from repeated injury and defective repair of alveolar epithelium, often in genetically predisposed individuals. 2. Cause of injury is obscure; various sources proposed, including chronic gastroesophageal reflux. 3. Genetic studies: germ line mutations leading to loss of telomerase associated with increased risk, suggesting cellular senescence contributes to a profibrotic phenotype. 4. IPF is a disorder of older adults, rarely occurring before the age of 55 years. 5. Genetic variant in the MUC5B gene altering mucin production found in approximately 35% of affected individuals, and some have germ line mutations in surfactant genes. 6. Hypothesized that abnormal epithelial repair at sites of chronic injury and inflammation leads to fibroblastic or myofibroblastic proliferation; excessive activation of profibrotic factors such as TGF-β may play a role.

Morphology of Pneumocystis jiroveci Infection in the Lung (4)

1. Involved areas of the lung contain a characteristic intraalveolar foamy, pink-staining exudate with hematoxylin-eosin (H&E) stain ("cotton candy" exudate). 2. The septa are thickened by edema and a minimal mononuclear infiltrate. 3. Special stains are required to visualize the organism. 4. Silver stain of tissue sections reveals round- to cup-shaped cysts (4 to 10 μm in diameter) within the alveolar exudates.

Klebsiella pneumoniae: Association with Gram-Negative Bacterial Pneumonia and Characteristics (3)

1. K. pneumoniae is the most frequent cause of gram-negative bacterial pneumonia. 2. Klebsiella-related pneumonia frequently afflicts debilitated and malnourished individuals, particularly chronic alcoholics. 3. Thick and gelatinous sputum is characteristic because the organism produces an abundant viscid capsular polysaccharide, which the patient may have difficulty coughing up.

Morphology of Pulmonary Embolism: Size, Location, and Consequences (3)

1. Large emboli may embed in the main pulmonary artery or its major branches or lodge astride the bifurcation as a saddle embolus, leading to sudden death from hypoxia or acute right-sided heart failure (acute cor pulmonale) without morphologic alterations in the lung. 2. Smaller blood clots get stuck in medium and small pulmonary arteries. 3. If adequate circulation and bronchial arterial flow are present, the vitality (חיוניות) of the lung parenchyma is maintained, but alveolar hemorrhage may occur due to ischemic damage to the endothelial cells.

Legionella pneumophila: Legionnaire Disease, Pontiac Fever, Transmission, Risk Factors, Diagnosis (5)

1. Legionnaire disease: - Caused by Legionella pneumophila - Epidemic and sporadic forms of pneumonia - Symptoms: fever, chills, headache, and cough 2. Pontiac fever: - Self-limited upper respiratory tract infection - No pneumonic symptoms 3. Transmission: - Flourishes in artificial aquatic environments - Water-cooling towers and domestic water supplies are common sources - Mode of transmission: inhalation of aerosolized organisms or aspiration of contaminated drinking water 4. Risk factors: - Immunocompromised individuals - Individuals with predisposing conditions such as cardiac, renal, immunologic, or hematologic disease - Organ transplant recipients 5. Diagnosis: - Demonstration of Legionella antigens in urine - Positive fluorescent antibody test on sputum samples - Culture remains the standard diagnostic modality - PCR-based tests can be used on bronchial secretions in atypical cases.

Morphology of Lobar Pneumonia: Consolidation, Lobar Pneumonia Stages, and Pleural Reaction (5)

1. Lobar Pneumonia Stages: a. Congestion: lung is heavy, boggy, and red; vascular engorgement, intraalveolar fluid, few neutrophils, numerous bacteria b. Red hepatization: massive confluent exudation, neutrophils, red cells, and fibrin fill alveolar spaces; red, firm, airless lobe c. Gray hepatization: disintegration of red cells, persistence of fibrinosuppurative exudate; grayish-brown color d. Resolution: enzymatic digestion of exudate, resorption, ingestion by macrophages, expectoration, or organization by fibroblasts 2. Pleural Reaction: - Pleural fibrinous reaction in early stages if consolidation extends to the surface (pleuritis) - May resolve or undergo organization, leaving fibrous thickening or permanent adhesions

Bacterial pneumonia: 2 Pattern of anatomic distribution (2)

1. Lobular bronchopneumonia 2. Lobar pneumonia

Sarcoidosis: Skin Lesions, Ocular Involvement, Lacrimal Gland, Parotitis, Spleen, Liver, Bone Marrow, Hypercalcemia, and Hypercalciuria (9)

1. Skin lesions encountered in 25% of patients. - Erythema nodosum (hallmark of acute sarcoidosis): raised, red, tender nodules on anterior legs, sarcoidal granulomas uncommon. - Discrete painless subcutaneous nodules with abundant noncaseating granulomas. 2. Eye and lacrimal gland involvement in 20-50% of patients. - Ocular involvement: iritis or iridocyclitis (unilateral or bilateral), corneal opacities, glaucoma, and possible vision loss. - Posterior uveal tract affected with choroiditis, retinitis, and optic nerve involvement. - Lacrimal gland inflammation leading to suppression of lacrimation (sicca syndrome). 3. Unilateral or bilateral parotitis with painful parotid gland enlargement in <10% of patients. - Some develop xerostomia (dry mouth). - Combined uveoparotid involvement known as Mikulicz syndrome. 4. Spleen contains granulomas in 75% of cases, but may appear unaffected grossly. 5. Liver demonstrates microscopic granulomatous lesions in portal triads. 6. Sarcoid involvement of bone marrow reported in 40% of patients, rarely causing severe manifestations. 7. Hypercalcemia and hypercalciuria not related to bone destruction, but caused by increased calcium absorption due to active vitamin D production by macrophages in granulomas.

Lung Abscess: Definition, 6 Causative Organisms, and Mechanisms (5)

1. Lung abscess is a localized area of suppurative necrosis within the pulmonary parenchyma, resulting in one or more large cavities. 2. The causative organism may be introduced into the lung by any of the following mechanisms: a) Aspiration of infective material from carious teeth, infected sinuses or tonsils. - May occur during oral surgery, anesthesia, coma, or alcoholic intoxication, and in debilitated patients with depressed cough reflexes. b) Aspiration of gastric contents with infectious organisms from the oropharynx. c) Complication of necrotizing bacterial pneumonias, mycotic infections, or bronchiectasis also may lead to lung abscesses. - Particularly those caused by S. aureus, Streptococcus pyogenes, K. pneumoniae, Pseudomonas spp., and, rarely, type d) Bronchial obstruction, particularly with bronchogenic carcinoma obstructing a bronchus or bronchiole. - Impaired drainage, distal atelectasis, tumor, and aspiration of blood and tumor fragments all contribute to the development of abscesses e) Septic embolism from infective endocarditis of the right side of the heart. f) Hematogenous spread of bacteria in disseminated pyogenic infection. - This occurs most characteristically in staphylococcal bacteremia and often results in multiple lung abscesses 3. Anaerobic bacteria are present in almost all lung abscesses and are the exclusive isolates in 1/3 to 2/3 of cases. 4. Common anaerobes: - Prevotella, Fusobacterium, Bacteroides, Peptostreptococcus, and microaerophilic streptococci.

Lung Adenocarcinomas: Morphology, Growth Patterns, and Progression (7)

1. Lung adenocarcinomas are usually peripherally located, but may also occur closer to the hilum. 2. These tumors grow slowly, form smaller masses than other subtypes, but tend to metastasize widely at an early stage. 3. Adenocarcinomas can have various growth patterns, including: (1) Acinar (gland-forming) (2) Papillary, mucinous (often multifocal and can manifest as pneumonia-like consolidation) (3) Solid types. 4. The solid variant requires special stains to demonstrate intracellular mucin and confirm adenocarcinomatous differentiation. 5. The precursor of adenocarcinoma is Atypical Adenomatous Hyperplasia (AAH), thought to progress to Adenocarcinoma In Situ (AIS), minimally invasive adenocarcinoma, and invasive adenocarcinoma. 6. AAH appears as a well-demarcated focus of epithelial proliferation, with a diameter of 5 mm or less, composed of cuboidal to low-columnar cells that demonstrate nuclear hyperchromasia, pleomorphism, and prominent nucleoli. It is monoclonal and shares many molecular aberrations with adenocarcinomas, such as KRAS mutations. 7. Adenocarcinoma In Situ (AIS) - Often peripheral, presents as a single nodule of 3 cm or less, with growth along preexisting structures and preservation of alveolar architecture. - The tumor cells grow in a monolayer along the alveolar septa. - By definition, AIS doesn't demonstrate destruction of alveolar architecture or stromal invasion with desmoplasia, features that would merit the diagnosis of invasive adenocarcinoma.

Spread and Clinical Manifestations of Lung Cancer Subtypes: Subtypes spread, Invasion to nodes, Syndromes and Pancoast tumors (7)

1. Lung cancer subtypes often spread to lymph nodes in the carina, mediastinum, neck (scalene nodes), and clavicular regions, eventually disseminating to distant sites. 2. Involvement of the left supraclavicular node (Virchow node) is particularly characteristic and may indicate an occult primary tumor. 3. Advanced cancers may extend into the pleural or pericardial space, causing inflammation and effusions. 4. Compression or infiltration of the superior vena cava can lead to venous congestion or the vena caval syndrome. 5. Apical neoplasms may invade the brachial or cervical sympathetic plexus, causing severe pain (in the distribution of the ulnar nerve) or Horner syndrome (ipsilateral enophthalmos, ptosis, miosis, anhidrosis). This is known as Pancoast syndrome. 6. Pancoast tumors are often accompanied by destruction of the first and second ribs and sometimes the thoracic vertebrae. 7. Tumor-node-metastasis (TNM) categories are used to indicate the size and spread of the primary neoplasm.

Clinical Features, Prognosis, and Treatment of Lung Carcinomas (9)

1. Lung carcinomas are often insidious, spreading before symptoms appear. 2. Symptoms like chronic cough or expectoration (eject from the throat or lungs by coughing or hawking and spitting) may indicate localized, resectable disease. - However, hoarseness (צְרִידוּת), chest pain, superior vena cava syndrome, pericardial or pleural effusion, persistent segmental atelectasis or pneumonitis usually indicate a poor prognosis. 3. Metastasis can present with symptoms associated with the brain, liver, or bones. 4. Overall, squamous cell carcinoma and adenocarcinoma have a more favorable prognosis than small cell lung carcinoma (SCLC). - If detected early, cure is possible through lobectomy or pneumonectomy. 5. Some unresectable adenocarcinomas with targetable mutations in tyrosine kinases like EGFR can respond remarkably to specific inhibitors. However, resistant tumors may eventually develop due to new mutations that alter the drug target or circumvent tumor dependence on it. 6. Immune checkpoint inhibitors are showing promise in treating smoking-related tumors. 7. SCLC generally has a poor prognosis. - By the time they are detected, they have usually spread, making surgical resection unviable. 8. Although SCLCs are sensitive to chemotherapy, they invariably recur. - Currently, targeted therapies are unavailable for SCLC. 9. The median survival even with treatment is only 1 year, with only 5% of patients alive at 10 years.

Pneumoconioses: Definition, Types, and Industries Associated with Dust Exposure (5)

1. Lung disorders caused by inhalation of mineral dusts 2. Term broadened to include diseases induced by organic and inorganic particulates, as well as chemical fume- and vapor-induced lung diseases 3. Three most common mineral dust pneumoconioses: (1) Coal dust (2) Silica (3) Asbestos 4. Usually stems from workplace exposure 5. Asbestos: risk for cancer extends to family members of asbestos workers and individuals exposed outside the workplace

Malignant Mesothelioma: Causes, Risk Factors, and Genetics

1. Malignant mesothelioma is a rare cancer of mesothelial cells, usually arising in the parietal or visceral pleura, and less commonly in the peritoneum and pericardium. 2. Approximately 80% to 90% of individuals with this cancer have a history of exposure to asbestos. Those who work directly with asbestos (shipyard workers, miners, insulators) are at greatest risk. 3. The latent period for developing malignant mesothelioma after initial exposure to asbestos is often 25 to 40 years. 4. Asbestos fibers, once inhaled, remain in the body for life, causing a lifetime risk that does not diminish over time. 5. Asbestos fibers can generate reactive oxygen species that cause DNA damage and mutations. Sequencing of mesothelioma genomes has revealed multiple driver mutations, many of which cluster in pathways involved in DNA repair, cell cycle control, and growth factor signaling. 6. One of the most commonly mutated genes in sporadic mesothelioma, BAP1, encodes a tumor suppressor involved in DNA repair that is also affected by germ line mutations in families showing a high incidence of mesothelioma.

Morphology and Patterns of Malignant Mesotheliomas (4)

1. Malignant mesotheliomas often begin with extensive pleural fibrosis and plaque formation, easily detected on computed tomography scans. 2. These tumors start in a localized area and, over time, spread widely either by contiguous growth or by diffuse seeding of pleural surfaces. 3. At autopsy, the affected lung typically is covered by a layer of yellow-white, firm, variably gelatinous tumor that obliterates the pleural space. 4. Mesotheliomas can directly invade the thoracic wall or subpleural lung tissue, but distant metastases are uncommon. They conform to one of three morphologic appearances: Epithelial: Cuboidal cells with small papillary buds line tubular and microcystic spaces. This is the most common pattern and the one most likely to be confused with a pulmonary adenocarcinoma. Sarcomatous: Spindled, occasionally fibroblastic-appearing cells grow in sheets. Biphasic: Contains both sarcomatous and epithelial areas.

Clinical Features and Manifestations of Cryptococcosis (5)

1. Manifests as pulmonary, central nervous system (CNS), or disseminated disease 2. Acquired by inhalation of aerosolized contaminated soil or bird droppings 3. Initially localizes in the lungs, then disseminates to other sites, especially the meninges 4. Variable tissue response, ranging from minimal inflammation to granulomatous reaction 5. In CNS, grows in gelatinous masses within the meninges or expands perivascular Virchow-Robin spaces, producing "soap-bubble lesions"

Morphological Changes Associated with Pulmonary Hypertension: Medial Hypertrophy, Atherosclerosis, and Plexiform Lesions (3)

1. Medial hypertrophy of pulmonary muscular and elastic arteries, pulmonary arterial atherosclerosis, and right ventricular hypertrophy. 2. Presence of organizing or recanalized thrombi suggests recurrent pulmonary emboli as the cause 3. Plexiform lesions: - Uncommon but characteristic change in pulmonary hypertension - A tuft of capillary formations producing a network or web that spans the lumens of dilated thin-walled, small arteries and may extend outside the vessel

Morphology of Pulmonary Infarction: Characteristics, Location, and Progression (7)

1. More likely with compromised cardiovascular status. 2. Mostly affect lower lobes and more than half are multiple. 3. Wedge-shaped with the base at the pleural surface and apex pointing toward the hilus. 4. Hemorrhagic and red-blue in early stages with Raised areas of coagulative necrosis. 5. Adjacent pleural surface often covered by a fibrinous exudate. 6. Red cells begin to lyse within 48 hours and Infarct becomes red-brown as hemosiderin is produced. 7. Fibrous replacement begins at the margins, eventually converting the infarct into a scar.

Pulmonary Embolism, Hemorrhage, and Infarction: Causes, Incidence, and Risk Factors (7)

1. More than 95% of all pulmonary emboli arise from thrombi within the large deep veins of the legs, particularly those involving the popliteal vein and larger veins above it. 2. Risk factors for venous thrombosis in the legs include: a. Prolonged bed rest with immobilization of the legs b. Surgery, especially orthopedic surgery on the knee or hip c. Severe trauma (including burns or multiple fractures) d. Congestive heart failure e. In women, the period around parturition or use of oral contraception pills with high estrogen content f. Disseminated cancer g. Primary disorders of hypercoagulability (e.g., factor V Leiden)

Clinical Manifestations and Prognosis of Carcinoid Tumors (5)

1. Most carcinoid tumors manifest with symptoms related to their intraluminal growth, including cough, hemoptysis, and recurrent bronchial and pulmonary infections. 2. Peripheral tumors are often asymptomatic and are discovered incidentally on chest radiographs. 3. Rarely, pulmonary carcinoids induce the Carcinoid syndrome, characterized by intermittent attacks of diarrhea, flushing, and cyanosis. 4. The reported 5- and 10-year survival rates for typical carcinoids are above 85%. 5. Survival rates for atypical carcinoids drop to 56% and 35% for 5- and 10-year survival, respectively.

Opportunistic Molds: Mucormycosis and Invasive Aspergillosis in Immunocompromised Hosts (3)

1. Mucormycosis and invasive aspergillosis are uncommon infections 2. Limited to immunocompromised hosts, particularly: a. Those with hematolymphoid malignancies and profound neutropenia b. Those undergoing corticosteroid therapy c. Hematopoietic stem cell transplant recipients

Morphology of Mucormycosis and Aspergillosis (4)

1. Mucormycosis caused by Zygomycetes class fungi, such as Rhizopus and Mucor 2. Zygomycetes hyphae are nonseptate and branch at right angles, while Aspergillus hyphae are septate and branch at acute angles 3. Both cause a nondistinctive, suppurative, sometimes granulomatous reaction 4. Predilection for invading blood vessel walls, causing hemorrhage, vascular necrosis, and infarction

Etiology and Pathogenesis of Tuberculosis: Mycobacteria, Transmission, and Mycobacterium Avium Complex (3)

1. Mycobacteria are slender rods that are acid-fast and have a high content of complex lipids. M. tuberculosis hominis is responsible for most cases of tuberculosis. 2. Transmission usually occurs through inhalation of airborne organisms in aerosols generated by expectoration or exposure to contaminated secretions of infected individuals. 3. Mycobacterium avium complex is much less virulent than M. tuberculosis and rarely causes disease in immunocompetent individuals but can cause disease in 10% to 30% of patients with AIDS.

Mycoplasma pneumoniae: Epidemiology and Diagnosis (3)

1. Mycoplasma infections are common among children and young adults. 2. They occur sporadically or as local epidemics in closed communities such as schools, military camps, and prisons. 3. Tests for Mycoplasma antigens and polymerase chain reaction (PCR) testing for Mycoplasma DNA are available.

Nasopharyngeal Carcinoma: Causes, Variants, and Treatment (5)

1. Nasopharyngeal carcinoma is a rare cancer with strong epidemiologic links to the Epstein-Barr virus (EBV) and a high frequency among the Chinese, suggesting viral oncogenesis on a genetic susceptibility background. 2. EBV is thought to first infect the nasopharyngeal epithelium and then nearby tonsillar B lymphocytes, leading to transformation of the epithelial cells. 3. The three histologic variants are: (1) keratinizing squamous cell carcinoma (2) nonkeratinizing squamous cell carcinoma (3) undifferentiated carcinoma. - The undifferentiated type is the most common and the one most closely linked with EBV. 4. Nasopharyngeal carcinomas invade locally, spread to cervical lymph nodes, and then metastasize to distant sites. 5. They tend to be radiosensitive, with 5-year survival rates of 50% reported, even for patients with advanced cancers. Responses to immune checkpoint inhibitors have been reported, providing a new therapeutic strategy for tumors unresponsive to conventional therapy.

Other Fibrosing Diseases: Differential Diagnosis of IPF, Features, Associations, and Prognosis (3)

1. Nonspecific Interstitial Pneumonia (NSIP): - Chronic bilateral interstitial lung disease - Frequent association with collagen vascular disorders (e.g., rheumatoid arthritis) - Better prognosis than IPF - Characterized by mild to moderate interstitial chronic inflammation and/or fibrosis, which is patchy but uniform in the areas involved. 2. Cryptogenic Organizing Pneumonia: - Presents with cough and dyspnea - Chest radiographs show subpleural or peribronchial patchy areas of airspace consolidation due to intraalveolar plugs of loose organizing connective tissue 3. Rheumatologic Entities: - Systemic sclerosis, rheumatoid arthritis, and systemic lupus erythematosus may be complicated by diffuse pulmonary fibrosis

Hospital-Acquired Pneumonias: Definition, Impact, Risk Factors, and Common Pathogens (3)

1. Nosocomial, or hospital-acquired, pneumonias are Pulmonary infections acquired during a hospital stay. 2. Risk Factors: Severe underlying disease, immunosuppression, prolonged antibiotic regimens, and mechanical ventilation (ventilator-associated pneumonia). 3. Common Pathogens: Gram-negative rods (Enterobacteriaceae and Pseudomonas spp.) and S. aureus; S. pneumoniae is not common in hospital settings.

Granulomatosis and Polyangiitis (GPA): Pulmonary Features, Symptoms, and Laboratory Findings (5)

1. Over 80% of patients develop upper-respiratory or pulmonary manifestations 2. Lung lesions characterized by necrotizing vasculitis and parenchymal necrotizing granulomatous inflammation 3. Upper-respiratory tract symptoms: chronic sinusitis, epistaxis, nasal perforation 4. Lung symptoms: cough, hemoptysis, chest pain 5. PR3-ANCAs (anti-neutrophil cytoplasmic antibodies) present in about 95% of cases

Pseudomonas aeruginosa Infection: Associations and Manifestations (3)

1. P. aeruginosa infection is commonly seen in nosocomial settings and can cause pneumonia in patients who are neutropenic, victims of extensive burns, and those requiring mechanical ventilation. 2. P. aeruginosa has a propensity to invade blood vessels at the site of infection, with consequent extrapulmonary spread. 3. Pseudomonas bacteremia is a fulminant disease, and histologic examination reveals organisms invading the walls of necrotic blood vessels (Pseudomonas vasculitis), leading to secondary coagulative necrosis of the pulmonary parenchyma.

Paraneoplastic Syndromes Associated with Lung Cancer (7)

1. Paraneoplastic syndromes are estimated to occur in 3% to 10% of patients with lung cancer. 2. Hypercalcemia: Due to secretion of a parathyroid hormone-related peptide, most often associated with squamous cell neoplasms. 3. Cushing syndrome: Resulting from increased production of adrenocorticotropic hormone (ACTH). 4. Syndrome of inappropriate secretion of anti-diuretic hormone (SIADH). 5. Neuromuscular syndromes: Includes Myasthenic syndrome, Peripheral neuropathy, and Polymyositis, often associated with small cell neoplasms. 6. Finger clubbing and hypertrophic pulmonary osteoarthropathy. 7. Coagulation abnormalities: - Includes Migratory thrombophlebitis, Nonbacterial endocarditis, and Disseminated intravascular coagulation (DIC), typically associated with adenocarcinomas.

Hypersensitivity Pneumonitis: Morphology (4)

1. Patchy mononuclear cell infiltrates in the pulmonary interstitium, with peribronchiolar accentuation. 2. Lymphocytes predominate, but plasma cells and epithelioid macrophages also are present. 3. "Loose," poorly formed granulomas, without necrosis, are present in more than two-thirds of cases, usually in a peribronchiolar location (Fig. 13.22). 4. Usually present in acute forms of hypersensitivity pneumonitis.

Morphologic Patterns in Viral Pneumonias: Macroscopic and Histologic Features, and Outcomes (7)

1. Patchy or whole lobes involved, bilaterally or unilaterally 2. Red-blue, congested, and subcrepitant areas 3. Inflammatory reaction largely confined to alveolar walls 4. Widened, edematous septa with mononuclear inflammatory infiltrate (lymphocytes, macrophages, and occasionally plasma cells) 5. Alveolar spaces usually free of cellular exudate 6. Severe cases may have diffuse alveolar damage with hyaline membranes 7. Uncomplicated cases: subsidence of disease followed by reconstitution of normal architecture

Secondary Tuberculosis: Relationship with HIV Infection in relation to CD4+ count (2)

1. Patients with less severe immunosuppression (CD4+ T cell counts >300 cells/μL) present with "usual" secondary tuberculosis (apical disease with cavitation) 2. Patients with more advanced immunosuppression (CD4+ T cell counts below 200 cells/μL) present with a clinical picture that resembles progressive primary tuberculosis (lower and middle lobe consolidation, hilar lymphadenopathy, and noncavitary disease)

Pathogenesis of Asbestos-Related Diseases: Inflammasome Activation, Fibrosis, Oncogenic Effects, and Synergy with Tobacco Smoke (4)

1. Phagocytosed asbestos fibers activate the inflammasome and damage phagolysosomal membranes, stimulating the release of proinflammatory factors and fibrogenic mediators. 2. Asbestos may function as both a tumor initiator and a promoter. 3. Oncogenic effects of asbestos on the mesothelium are mediated by reactive free radicals generated by asbestos fibers, which preferentially localize in the distal lung close to the mesothelial layer. 4. Adsorption of carcinogens in tobacco smoke onto asbestos fibers contributes to the remarkable synergy between tobacco smoking and the development of lung carcinoma in asbestos workers.

Pleural Effusion and Pleuritis: Types, Causes, Consequences

1. Pleural effusions can be either transudates or exudates. 2. Hydrothorax is where the effusion is a TRANSUDATE, with Congestive heart failure being the most common cause of bilateral hydrothorax. 3. An Exudate, characterized by protein content greater than 30 g/L and often inflammatory cells, suggests pleuritis. 4. The four principal causes of PLEURAL EXUDATE formation are: (1) microbial invasion (2) cancer (lung carcinoma, metastatic neoplasms, mesothelioma) (3) pulmonary infarction (4) viral pleuritis. 5. Less common causes of exudative pleural effusions include systemic lupus erythematosus, rheumatoid arthritis, uremia, and previous thoracic surgery. 6. Malignant effusions are usually large and frequently bloody (hemorrhagic pleuritis), and cytologic examination may reveal the malignant cells. 7. Transudates and serous exudates are usually resorbed without residual effects if the inciting cause is controlled or remits. 8. Fibrinous, hemorrhagic, and suppurative exudates may lead to fibrous organization, yielding adhesions or fibrous pleural thickenings that may undergo calcification.

Pneumocytosis: Pneumocystis jiroveci: An Opportunistic Fungal Infection in Immunocompromised Individuals (7)

1. Pneumocystis jiroveci, previously considered a protozoan, is now classified as a fungus. 2. Almost all individuals are exposed to Pneumocystis during the first few years of life, but infection usually remains latent. 3. Reactivation occurs almost exclusively in immunocompromised individuals, such as those with AIDS, malnourished infants, and immunosuppressed individuals (e.g., organ transplant recipients, those receiving cytotoxic chemotherapy, or corticosteroids). 4. In patients with AIDS, the risk for acquiring P. jiroveci infection increases in inverse proportion to the CD4+ T cell count, with counts less than 200 cells/μL having strong predictive value. 5. Pneumocystis infection is largely confined to the lung, where it produces an interstitial pneumonitis.

Diagnosis and Management of Pneumocystis Pneumonia (5)

1. Pneumocystis pneumonia should be considered in any immunocompromised patient with respiratory symptoms and abnormal findings on chest radiograph. 2. Fever, dry cough, and dyspnea occur in 90% to 95% of patients, with typical radiographic evidence of bilateral perihilar and basilar infiltrates. 3. Hypoxia is frequent, and pulmonary function studies show a restrictive lung defect. 4. The most sensitive and effective method of diagnosis is to identify the organism in induced sputum or bronchoalveolar lavage fluid using immunofluorescence. 5. If treatment is initiated before widespread involvement, the outlook for recovery is good.

Pulmonary Infections: Pneumonia, Factors Contributing to Infections, and Clinical Settings (5)

1. Pneumonia: infection in the lung, responsible for one-sixth of all deaths in the US 2. Lung vulnerability to infection due to (1) many microbes are airborne and readily inhaled into the lungs (2) nasopharyngeal flora are regularly aspirated during sleep, even by healthy individuals (3) lung diseases often lower local immune defenses. 3. Increased infections in patients with defects in innate immunity (neutrophil and complement defects) or adaptive immunity (humoral immunodeficiency) 4. Lifestyle choices (e.g., smoking and alcohol) can interfere with host immune defense mechanisms 5. Bacterial pneumonias classified by etiologic agent or clinical setting

Pneumothorax, Hemothorax, and Chylothorax: Definitions, Causes, and Complications

1. Pneumothorax - Refers to the presence of air or other gas in the pleural sac. - It can be simple (or spontaneous) or secondary due to thoracic or lung disorders such as emphysema, lung abscess, tuberculosis, carcinoma, or a fractured rib. 2. Secondary pneumothorax results from the rupture of any pulmonary lesion situated close to the pleural surface that allows inspired air to enter the pleural cavity. 3. Complications of pneumothorax include tension pneumothorax, lung scarring that prevents re-expansion, hydropneumothorax due to fluid collection in the pleural cavity, and empyema (pyopneumothorax). 4. Hemothorax - Collection of whole blood in the pleural cavity - Can be a complication of a ruptured intrathoracic aortic aneurysm. - Unlike bloody pleural effusions, the blood clots within the pleural cavity in hemothorax. 5. Chylothorax - Pleural collection of milky lymphatic fluid containing microglobules of lipid, usually caused by obstruction of major lymph ducts, typically by an intrathoracic cancer.

Primary Tuberculosis: Definition, Consequences, and Factors Influencing Disease Progression (5)

1. Primary tuberculosis develops in previously unexposed and unsensitized patients, with 5% acquiring significant disease. 2. In most healthy individuals, primary tuberculosis results in foci of scarring that may harbor viable bacilli and serve as a nidus for disease reactivation if host defenses wane. - the foci of scarring in primary tuberculosis can act as a starting point or focus for the disease to reactivate if the immune system weakens. 3. Progressive primary tuberculosis occurs in immunocompromised patients or those with subtle defects in host defenses, such as malnourished individuals or certain racial groups like the Inuit. 4. Incidence of progressive primary tuberculosis is high in HIV-positive patients with significant immunosuppression (CD4+ T cell counts below 200 cells/μL). 5. Immunosuppression results in an inability to mount a CD4+ T cell-mediated response, leading to a lack of tissue hypersensitivity reaction and absence of characteristic caseating granulomas (nonreactive tuberculosis).

Primary Tuberculosis Morphology: Ghon Focus, Ghon Complex, and Ranke Complex (3)

1. Primary tuberculosis typically begins in the lungs, in the distal air spaces of the lower part of the upper lobe or upper part of the lower lobe. 2. Ghon focus: - A 1-cm to 1.5-cm area of gray-white inflammatory consolidation with caseous necrosis at the center. 3. Ghon complex (Fiugre): - Combination of parenchymal and nodal lesions, where bacilli travel via lymphatic vessels to regional lymph nodes, which also often caseate. 4. Ranke complex: - Calcified Ghon complex detectable on radiograph after the development of cell-mediated immunity, which controls the infection in about 95% of cases.

Clinical Features of Pulmonary Hypertension and Treatment Options (7)

1. Produce symptoms in advanced disease 2. Most common in women 20-40 years old 3. Occurs occasionally in young children 4. Presenting features: dyspnea, fatigue, anginal chest pain 5. Over time, respiratory distress, cyanosis, and right ventricular hypertrophy appear, and death from decompensated cor pulmonale, often with superimposed thromboembolism and pneumonia, ensues within 2 to 5 years in 80% of patients. 6. For those with secondary disease, therapy is directed at the trigger (e.g., thromboembolic disease or hypoxemia). 7. A variety of vasodilators have been used with varying success in those with group 1 disease or refractory disease belonging to other groups.

Clinical Features of Lung Abscesses: Symptoms, Association with Bronchogenic Carcinoma (4)

1. Prominent cough with copious amounts of foul-smelling, purulent or sanguineous sputum, spiking fever, malaise, clubbing of the fingers, weight loss, and anemia. 2. Hemoptysis may occasionally occur. 3. Abscesses occur in 10-15% of patients with bronchogenic carcinoma, and underlying carcinoma must be considered in older adults with a suspected lung abscess. 4. Secondary amyloidosis may develop in chronic cases.

Morphology of Anthracosis, Simple CWP, and Complicated CWP (3)

1. Pulmonary anthracosis: - Most innocuous coal-induced pulmonary lesion - Carbon pigment engulfed by alveolar or interstitial macrophages, accumulating in connective tissue along pulmonary and pleural lymphatics and draining lymph nodes 2. Simple coal worker's pneumoconiosis (CWP): - Characterized by coal macules and larger coal nodules - Dust-laden macrophages and small amounts of collagen fibers in a delicate network - Upper lobes and upper zones of lower lobes more heavily involved - Centrilobular emphysema may occur 3. Complicated CWP or progressive massive fibrosis (PMF): - Develops on the background of simple CWP by coalescence of coal nodules - Characterized by multiple, dark black scars larger than 2 cm, sometimes up to 10 cm in diameter, consisting of dense collagen and pigment

Pulmonary Hypertension: Subtypes (5)

1. Pulmonary arterial hypertension: - Heritable forms - Affects small pulmonary muscular arterioles - Associated with connective tissue diseases, HIV, and congenital heart disease with left to right shunts 2. Pulmonary hypertension due to left-sided heart disease: - Involves systolic and diastolic dysfunction and valvular disease 3. Pulmonary hypertension due to lung diseases and/or hypoxia: - Includes COPD and interstitial lung disease 4. Chronic thromboembolic pulmonary hypertension 5. Pulmonary hypertension with unclear or multifactorial mechanisms

Idiopathic Pulmonary Fibrosis (IPF): Characteristics, Epidemiology, and Histologic Pattern (5)

1. Pulmonary disorder of unknown etiology characterized by patchy, progressive bilateral interstitial fibrosis. 2. Also known as Cryptogenic Fibrosing Alveolitis. 3. Males affected more often than females; disease of aging, rarely occurring before 50 years of age. 4. Radiologic and histologic pattern of fibrosis referred to as usual interstitial pneumonia (UIP), required for IPF diagnosis. 5. IPF is a diagnosis of exclusion; similar pathologic changes may be present in asbestosis, collagen vascular diseases, and other conditions.

Clinical Features of Viral Pneumonia: Presentation, Symptoms (3)

1. Ranges from undiagnosed severe upper-respiratory tract infection to fulminant, life-threatening infection in immunocompromised patients 2. Initial presentation: Acute, nonspecific febrile illness with fever, headache, malaise, and later cough with minimal sputum 3. Ventilation-perfusion mismatch: - Localization of inflammatory exudate to alveolar walls prevents oxygenation of blood, causing mismatch of ventilation and perfusion - Respiratory distress may seem out of proportion to physical and radiographic findings

Complications of pneumonia: (3)

(1) Tissue destruction and necrosis, causing abscess formation (2) Spread of infection to the pleural cavity, causing the intrapleural fibrinosuppurative reaction known as empyema (3) Bacteremic dissemination to the heart valves, pericardium, brain, kidneys, spleen, or joints, causing metastatic abscesses, endocarditis, meningitis, or suppurative arthritis

ACUTE RESPIRATORY DISTRESS SYNDROME (ARDS) Clinical features: Factors increasing the risk of poor prognosis

(1) advanced age (2) bacteremia (sepsis) (3) development of multiorgan failure.

OBSTRUCTIVE LUNG (AIRWAY) DISEASES: Asthma The 4 hallmarks of asthma

(1) intermittent, reversible airway obstruction (2) chronic bronchial inflammation with eosinophils (3) bronchial smooth muscle cell hypertrophy and hyperreactivity (4) increased mucus secretion.

Lung diseases: Classification based on Structure affected

(1) the airways (2) the interstitium (3) the pulmonary vascular system.

ACUTE RESPIRATORY DISTRESS SYNDROME (ARDS) Morphology: Diffuse alveolar damage in the acute phase

(A) Diffuse alveolar damage in the acute phase. Some alveoli are collapsed, while others are distended; many are lined by bright pink hyaline membranes (arrow)

ACUTE RESPIRATORY DISTRESS SYNDROME (ARDS) Morphology: Diffuse alveolar damage - Healing stage

(B) The healing stage is marked by resorption of hyaline membranes and thickening of alveolar septa by inflammatory cells, fibroblasts, and collagen. Numerous reactive type II pneumocytes also are seen at this stage (arrows), associated with regeneration and repair.

Table: Emphysema - Anatomic Site - Major Pathologic Changes - Etiology - Signs/Symptoms

- Anatomic Site : Acinus - Major Pathologic Changes : a. Air space enlargement b. Wall destruction - Etiology : Tobacco smoke - Signs/Symptoms : Dyspnea

Table: Small airway disease, bronchiolitis* - Anatomic Site - Major Pathologic Changes - Etiology - Signs/Symptoms

- Anatomic Site : Bronchioles - Major Pathologic Changes : a. Inflammatory scarring b. Partial obliteration of bronchioles - Etiology : Tobacco smoke - Signs/Symptoms : Dyspnea and cough

Table: Asthma - Anatomic Site - Major Pathologic Changes - Etiology - Signs/Symptoms

- Anatomic Site : Bronchus - Major Pathologic Changes : a. Smooth muscle hypertrophy and hyperplasia b. Excessive mucus c. Inflammation - Etiology : Immunologic or undefined causes - Signs/Symptoms : Episodic wheezing, cough, dyspnea

Table: Bronchiectasis - Anatomic Site - Major Pathologic Changes - Etiology - Signs/Symptoms

- Anatomic Site : Bronchus - Major Pathologic Changes : Airway dilation and scarring - Etiology : Persistent or severe infections - Signs/Symptoms : Cough, purulent sputum, fever

Table: Chronic Bronchitis: - Anatomic Site - Major Pathologic Changes - Etiology - Signs/Symptoms

- Anatomic Site : Bronchus - Major Pathologic Changes: a. Mucous gland hypertrophy and hyperplasia, b. hypersecretion - Etiology : Tobacco smoke, air pollutants - Signs/Symptoms : Cough, sputum production

OBSTRUCTIVE LUNG (AIRWAY) DISEASES: Bronchiectasis Clinical features

- Bronchiectasis is characterized by severe, persistent cough associated with expectoration of mucopurulent, sometimes fetid (מַסרִיחַ), sputum. - Other common symptoms: dyspnea, rhinosinusitis, and hemoptysis. - Symptoms often are episodic and are precipitated by upper respiratory tract infections or the introduction of new pathogenic agents - Severe, widespread bronchiectasis may lead to significant obstructive ventilatory defects, with hypoxemia, hypercapnia, pulmonary hypertension, and cor pulmonale

OBSTRUCTIVE LUNG (AIRWAY) DISEASES: Bronchiectasis Morphology: - Bronchiectasis usually affects the - Usually, the most severe involvement is found in the - Airway dilation

- Bronchiectasis usually affects the lower lobes bilaterally, particularly those air passages that are most vertical. - Usually, the most severe involvement is found in the more distal bronchi and bronchioles. - The airways may be dilated to as much as four times their usual diameter and can be seen on gross examination almost out to the pleural surface (Fig. 13.12). The histologic findings vary with the activity and chronicity of the disease. - In full-blown active cases, an intense acute and chronic inflammatory exudate within the walls of the bronchi and bronchioles leads to desquamation of lining epithelium and extensive areas of ulceration - Typically, mixed flora are found in the sputum

Clinical Features of Candidiasis Infections (6):

- Candidiasis can involve the mucous membranes, skin, and deep organs (invasive candidiasis). 1. Oral thrush: - Most common presentation - Gray-white, dirty-looking pseudomembranes on oral mucosa - Associated with newborns, debilitated patients, oral corticosteroids, broad-spectrum antibiotics, and HIV-positive patients 2. Vaginitis: - Common in women, especially diabetics, pregnant women, or those on oral contraceptives 3. Esophagitis: - Common in patients with AIDS and hematolymphoid malignancies - Presents with dysphagia and retrosternal pain 4. Skin infection: - Various forms: onychomycosis, paronychia, folliculitis, intertrigo, balanitis, and diaper rash 5. Chronic mucocutaneous candidiasis: - Affects mucous membranes, skin, hair, and nails - Associated with various T cell defects, including Job syndrome 6. Invasive candidiasis: - Blood-borne dissemination to various tissues or organs - Common patterns include renal abscess, myocardial abscess and endocarditis, brain involvement, endophthalmitis, hepatic abscesses, and Candida pneumonia

ATELECTASIS (COLLAPSE) • Contraction atelectasis - Second name - Occurs when

- Cicatrization atelectasis - Occurs when local or diffuse fibrosis affecting the lung or the pleura hamper lung expansion.

Alveolar epithelium cells

- Consisting of a continuous layer of two principal cell types: (1) type I pneumocytes fattened, plate-like covering 95% of the alveolar surface; and rounded (2) type II pneumocytes synthesize pulmonary surfactant and are the main cell type involved in repair of alveolar epithelium after damage to type I pneumocytes.

OBSTRUCTIVE LUNG (AIRWAY) DISEASES: Emphysema: - Clinical features

- Dyspnea usually is the first symptom - It begins insidiously but is steadily progressive - In patients with underlying chronic bronchitis or chronic asthmatic bronchitis, cough and wheezing may be the initial complaints - Weight loss is common and may be severe enough to suggest an occult (נִסתָר) malignant tumor

OBSTRUCTIVE PULMONARY DISEASES: - Main Etiologies of diffuse obstructive disorders

- Expiratory obstruction may result from: (1) anatomic airway narrowing, classically observed in asthma, (2) loss of elastic recoil, characteristic of emphysema. This is almost certainly because cigarette smoking is the major underlying cause of both

OBSTRUCTIVE LUNG (AIRWAY) DISEASES: Asthma Morphology

- In fatal cases, the lungs are distended due to air trapping (overinflation), and there may be small areas of atelectasis. - The most striking finding is occlusion of bronchi and bronchioles by thick, tenacious mucous plugs containing whorls of shed epithelium (Curschmann spirals) - Numerous eosinophils and Charcot-Leyden crystals (crystalloids made up of the eosinophil protein galectin-10) also are present. - Airway remodeling: • Thickening of airway wall • Sub-basement membrane fibrosis (Fig. 13.11) • Increased submucosal vascularity • An increase in size of the submucosal glands and goblet cell metaplasia of the airway epithelium • Hypertrophy and/or hyperplasia of the bronchial muscle

ACUTE RESPIRATORY DISTRESS SYNDROME (ARDS) Morphology: Acute phase of ARDS Other than HM

- Lungs are dark red, firm, airless, and heavy - Capillary congestion - Necrosis of alveolar epithelial cells - Interstitial and Intraalveolar edema and hemorrhage - (particularly with sepsis) Collections of neutrophils in capillaries

OBSTRUCTIVE LUNG (AIRWAY) DISEASES: Asthma Occupational Asthma - Triggered by - Asthma attacks usually develop after

- Occupational asthma may be triggered by fumes (epoxy resins, plastics), organic and chemical dusts (wood, cotton, platinum), gases (toluene), and other chemicals. - Asthma attacks usually develop after repeated exposure to the inciting antigen(s).

OBSTRUCTIVE LUNG (AIRWAY) DISEASES: Emphysema: (1) Centriacinar (centrilobular) emphysema - Main feature

- The distinctive feature of centriacinar emphysema is that the central or proximal parts of the acini, formed by respiratory bronchioles, are affected, while distal alveoli are spared. Thus, both emphysematous and normal air spaces exist within the same acinus and lobule In severe centriacinar emphysema, the distal acinus also becomes involved, and thus, the differentiation from panacinar emphysema becomes difficult. - This type of emphysema is most common in cigarette smokers, often in association with chronic bronchitis.

OBSTRUCTIVE LUNG (AIRWAY) DISEASES: Chronic Bronchitis Main features

- The distinctive feature of chronic bronchitis is hypersecretion of mucus, beginning in the large airways.

ACUTE RESPIRATORY DISTRESS SYNDROME (ARDS) Diffuse Alveolar Damage (DAD) - Definition - Components of HM - Pathophysiology - Relation to ARDS

- The histologic manifestation of ARDS in the lungs is known as diffuse alveolar damage (DAD). An acute lung condition with the presence of hyaline membranes. - These hyaline membranes are made up of dead cells, surfactant, and proteins. - The hyaline membranes deposit along the walls of the alveoli, where gas exchange typically occurs, thereby making gas exchange difficult. - It is important to note that DAD can be seen in situations other than ARDS (such as acute interstitial pneumonia) and that ARDS can occur without DAD.

OBSTRUCTIVE LUNG (AIRWAY) DISEASES: Emphysema: (1) Centriacinar (centrilobular) emphysema - Main location of damage

- The lesions are more common and severe in the upper lobes, particularly in the apical segments

OBSTRUCTIVE PULMONARY DISEASES: - The major diffuse obstructive disorders

- The major diffuse obstructive disorders are (1) emphysema (2) chronic bronchitis (3) bronchiectasis (4) asthma It should be noted that emphysema is defined on the basis of morphologic and radiologic features, whereas chronic bronchitis is defined on the basis of clinical features

Pathogenesis of Tuberculosis: Steps, Immune Response, and Tissue Damage (5)

- The pathologic features of tuberculosis, such as caseating granulomas and cavitation, are the result of the destructive tissue hypersensitivity that is part and parcel of the host immune response - The sequence of events from inhalation of the infectious inoculum to containment of the primary focus can be outlined as follows: 1. Entry into macrophages: Mycobacteria enter macrophage endosomes, mediated by macrophage mannose receptor and complement receptors. 2. Replication in macrophages: Mycobacterium inhibits lysosome-phagocytic vacuole fusion, allowing persistence and proliferation within pulmonary alveolar macrophages. 3. Development of cell-mediated immunity: Occurs approximately 3 weeks after exposure, leading to the generation of TH1 CD4+ T cells that secrete IFN-γ. 4. T cell-mediated macrophage activation: IFN-γ activates macrophages, which release TNF, iNOS, and anti-microbial peptides to combat mycobacteria. 5. Granulomatous inflammation and tissue damage: TH1 response leads to the formation of granulomas and caseous necrosis; activated macrophages secrete TNF and chemokines to recruit more monocytes.

Clinical Features of Rhinocerebral and Pulmonary Mucormycosis, Invasive Aspergillosis, Allergic Bronchopulmonary Aspergillosis, and Aspergilloma (4)

1. Rhinocerebral mucormycosis: - Zygomycetes colonize nasal cavity or sinuses - Spread to brain, orbit, and other head and neck structures - Diabetic ketoacidosis predisposes patients to fulminant invasive form 2. Pulmonary mucormycosis: - Localized (cavitary lesions) or diffuse (miliary involvement) 3. Invasive aspergillosis: - Preferentially localizes to lungs - Necrotizing pneumonia; often fatal if disseminated to the brain 4. Allergic bronchopulmonary aspergillosis: - Occurs in asthma patients - Type I hypersensitivity against Aspergillus - Circulating IgE antibodies against Aspergillus and peripheral eosinophilia 5. Aspergilloma: - Fungal colonization of preexisting pulmonary cavities - Ball valve effect predisposes to infection and hemoptysis

ATELECTASIS (COLLAPSE) • Compression atelectasis - Usually associated with - Most common cause - Etiology

- Usually associated with accumulation of fluid, blood, or air within the pleural cavity. - A frequent cause is pleural effusions occurring in the setting of congestive heart failure. - Leakage of air into the pleural cavity (pneumothorax) also leads to compression atelectasis

Carcinoma of the Larynx: Epidemiology, Pathology, Clinical Manifestations, (8)

1 Represents only 2% of all cancer and Occurs most commonly after 40 years of age and is more common in men than in women (gender ratio of 7:1). 2. Environmental influences, such as smoking, alcohol, and asbestos exposure, play important roles in its causation. 3. Human papillomavirus sequences detected in about 15% of tumors, which have a better prognosis. 4. About 95% of laryngeal cancers are typical squamous cell carcinomas. 5. Rarely, adenocarcinomas arise from mucous glands. 6. Tumors develop on the vocal cords (glottic tumors) in 60% to 75% of cases, above the cords (supraglottic) in 25% to 40% of cases, or below the cords (subglottic) in less than 5% of cases. 7. Glottic tumors are usually keratinizing, well to moderately differentiated squamous cell carcinomas. 8. Persistent hoarseness (צְרִידוּת) is a common clinical manifestation.

Tuberculosis (TB): Predisposition Factors (10)

1) Older adults, the urban poor (העניים העירוניים) 2) Patients with AIDS - HIV infection is the dominant risk factor for the development of tuberculosis. 3) Members of minority communities 4) Diabetes mellitus 5) Hodgkin lymphoma 6) Chronic lung disease (particularly silicosis) 7) Chronic renal failure 8) Malnutrition 9) Alcoholism 10) Immunosuppression

Morphology and Diagnostic Features of Cryptococcus Fungus (4)

1. 5- to 10-μm yeast with a thick, gelatinous capsule 2. Reproduces by budding, but no pseudohyphal or true hyphal forms 3. Capsule identification is key for diagnosis 4. India ink or periodic acid-Schiff staining highlights the fungus, and cryptococcal latex agglutination assay is positive in more than 95% of patients infected with the organism

Clinical Features of Community-Acquired Acute Bacterial Pneumonia (2)

1. Abrupt onset of high fever, shaking chills, and cough producing mucopurulent sputum 2. Occasional hemoptysis 3. Pleuritic pain and pleural friction rub when pleuritis is present 4. Whole lobe radiopaque in lobar pneumonia 5. Focal opacities in bronchopneumonia

Morphology of Lung Abscesses: Size, Localization, Number, and Complications (5)

1. Abscesses range in diameter from a few millimeters to large cavities 5 to 6 cm across. 2. Aspiration-induced abscesses: more common on the right side, single, and located in the posterior segment of the upper lobe or apical segments of the lower lobe. 3. Pneumonia or bronchiectasis-induced abscesses: multiple, basal, and diffusely scattered. 4. Septic emboli and hematogenous seeding-induced abscesses: multiple and may affect any region of the lungs. - Rupture into airways: can cause partial drainage, producing an air-fluid level on radiographic examination. 5. Complications: a) Rupture into the pleural cavity: may produce bronchopleural fistulas, pneumothorax, or empyema. b) Embolization of septic material to the brain: may cause meningitis or brain abscess. c) Histologic examination: suppurative focus is surrounded by variable amounts of fibrous scarring and mononuclear infiltration, depending on the chronicity of the lesion.

Clinical Features of Dimorphic Fungal Infections: Histoplasmosis, Coccidioidomycosis, and Blastomycosis (3)

1. Acute (primary) pulmonary infection: - Resembles a "flulike" syndrome - Self-limited - Associated with regional lymph node lesions 2. Chronic (granulomatous) pulmonary disease: - Predilection for the upper lobe - Manifestations may include cough, hemoptysis, dyspnea, and chest pain - May resemble bronchogenic carcinoma radiologically 3. Disseminated miliary disease (in infants or immunocompromised adults): - Focal collections of phagocytes containing yeast forms in liver, spleen, lymph nodes, gastrointestinal lymphoid tissue, and bone marrow - Adrenal glands and meninges may be involved - Symptoms include febrile illness, hepatosplenomegaly, anemia, leukopenia, and thrombocytopenia

Hypersensitivity Pneumonitis: Clinical Features

1. Acute Hypersensitivity Pneumonitis: Presents with fever, cough, dyspnea, and constitutional signs and symptoms arising 4 to 8 hours after exposure. 2. Chronic Hypersensitivity Pneumonitis: Characterized by insidious onset of cough, dyspnea, malaise, and weight loss. - Failure to remove the inciting agent from the environment results in an irreversible chronic interstitial pulmonary disease.

Streptococcus pneumoniae: Risk Factors, Diagnosis, and Vaccination (3)

1. Risk factors for pneumococcal infections: (1) Chronic diseases (CHF, COPD, diabetes) (2) Congenital or acquired defects in immunoglobulin production (e.g., AIDS) (3) Decreased or absent splenic function (e.g., sickle cell disease, splenectomy) 2. Diagnosis: - Numerous neutrophils in sputum containing gram-positive, lancet-shaped diplococci - Isolation of pneumococci from blood cultures (more specific but less sensitive) 3. Vaccination: - Pneumococcal vaccines containing capsular polysaccharides from common serotypes are used in high-risk individuals

Infections of the Upper Respiratory Tract: Epiglottitis, Laryngitis, and Croup - Causes, Manifestations, Complications (4)

1. Acute bacterial epiglottitis - Predominantly affects young children and - Caused by H. influenza. - The disease is characterized by pain and airway obstruction. - Vaccination against H. influenzae has greatly decreased the incidence of this disease. 2. Acute laryngitis - Can result from inhalation of irritants or allergic reactions, or it can be caused by the same agents that produce the common cold. - Two rare but serious forms are tuberculous laryngitis (usually a consequence of active tuberculosis) and diphtheritic laryngitis (caused by Corynebacterium diphtheriae and largely prevented by immunization). 3. In children, parainfluenza virus is the most common cause of laryngotracheobronchitis, also known as croup, but other agents such as respiratory syncytial virus can also cause this condition. - Croup can cause inspiratory stridor (high-pitched respiratory sound that can be assessed during breathing) and harsh (מתמשך), persistent cough, and in severe cases, can lead to respiratory failure. 4. Viral infections in the upper respiratory tract predispose the patient to secondary bacterial infection, particularly by staphylococci, streptococci, and H. Influenzae.

Pulmonary Eosinophilia: Definition, Categories, and Clinical Features (5)

1. Acute eosinophilic pneumonia: - Rapid onset of fever, dyspnea, hypoxia, and diffuse pulmonary infiltrates on chest radiographs. - Bronchoalveolar lavage fluid typically contains more than 25% eosinophils. - There is prompt response to corticosteroids. 2. Simple pulmonary eosinophilia (Loeffler syndrome): - Transient pulmonary lesions, eosinophilia in the blood, and a benign clinical course. - The alveolar septa are thickened by an infiltrate containing eosinophils and occasional giant cells. 3. Tropical eosinophilia caused by infection with microflariae and helminthic parasites. 4. Secondary eosinophilia seen in association with asthma, drug allergies, and certain forms of vasculitis. 5. Idiopathic chronic eosinophilic pneumonia: - Characterized by aggregates of lymphocytes and eosinophils within the septal walls and the alveolar spaces, typically in the periphery of the lung fields. - Accompanied by high fever, night sweats, and dyspnea. - This is a disease of exclusion, once other causes of pulmonary eosinophilia have been ruled out.

Infections of the Upper Respiratory Tract: Pharyngitis - Causes, Manifestations, Complications (3)

1. Acute pharyngitis - Manifesting as a sore throat, may be caused by several agents. - Mild pharyngitis often accompanies a common cold and is the most common form. - More severe forms with tonsillitis, associated with marked hyperemia and exudates, occur with β-hemolytic streptococcal and adenovirus infections. 2. Streptococcal tonsillitis - Important to recognize and treat early due to the associated potential for the development of peritonsillar abscesses ("quinsy") or for progression to poststreptococcal glomerulonephritis and acute rheumatic fever. 3. Other causes of pharyngitis include Coxsackievirus A infection, which can produce pharyngeal vesicles and ulcers (herpangina), and Epstein-Barr virus (EBV), which causes infectious mononucleosis (also known as the "kissing disease" due to its common mode of transmission in previously nonexposed individuals).

Acute Upper Respiratory Tract Infections: Common Cold - Clinical Features, Pathogens, Course (6)

1. Acute upper respiratory tract infections are common 2. Most frequently manifesting as the "common cold." 3. Symptoms include nasal congestion with watery discharge, sneezing, scratchy sore throat, and a slight increase in temperature that is more pronounced in young children. 4. The most common pathogens are rhinoviruses, but coronaviruses, respiratory syncytial viruses, parainfluenza and influenza viruses. 5. These infections most commonly occur in fall and winter and are usually self-limiting, lasting for 1 week or less. 6. In a minority of cases, colds may be complicated by the development of bacterial otitis media or sinusitis.

Coal Worker's Pneumoconiosis: Spectrum of Lung Findings, PMF, and Coal Rank (3)

1. Asymptomatic anthracosis: Pigment deposits without a cellular reaction 2. Simple coal worker's pneumoconiosis (CWP): Macrophage accumulation with little to no pulmonary dysfunction 3. Complicated CWP or progressive massive fibrosis (PMF): Extensive fibrosis and compromised lung function - Less than 10% of simple CWP cases progress to PMF

Clinical Features of Sarcoidosis: Presentation, Diagnosis, and Disease Course (7)

1. Asymptomatic in many individuals. Discovered on routine chest films or at autopsy. 2. Peripheral lymphadenopathy, cutaneous lesions, eye involvement, splenomegaly, or hepatomegaly may be presenting manifestations. 3. No definitive diagnostic test. 4. Tuberculosis must be excluded. 5. 65-70% recover with minimal or no residual manifestations. 6. 20% develop permanent lung dysfunction or visual impairment. 7. 10-15% succumb to progressive pulmonary fibrosis and cor pulmonale.

Moraxella catarrhalis: Epidemiology and Association with Diseases (3)

1. Bacterial Pneumonia: M. catarrhalis is being increasingly recognized as a cause of bacterial pneumonia, especially in older adults. 2. Acute Exacerbation of COPD: - M. catarrhalis is the second most common bacterial cause of acute exacerbation of COPD in adults. 3. Otitis Media: - M. catarrhalis, along with S. pneumoniae and H. influenza, is one of the three most frequent causes of otitis media in children.

Pulmonary Disease in HIV Infection: Common Pathogens, Noninfectious Diseases, and Role of CD4+ T Cell Count (3)

1. Bacterial lower-respiratory tract infection: - Common in HIV infection - Usual pathogens: S. pneumoniae, S. aureus, H. influenza, gram-negative rods - More common, severe, and often associated with bacteremia in HIV-infected individuals 2. Noninfectious diseases: - Kaposi sarcoma, pulmonary non-Hodgkin lymphoma, primary lung cancer - Occur with increased frequency in HIV-infected individuals 3. CD4+ T cell count in differential diagnosis: - Mildly suppressed CD4+ counts (more than 200 cells/μL): bacterial and tubercular infections - CD4+ counts below 200 cells/μL: Pneumocystis pneumonia - CD4+ counts below 50 cells/μL: CMV and M. avium complex infections

Hypersensitivity Pneumonitis: Immunologic Basis of Hypersensitivity Pneumonitis: Evidence (4)

1. Bronchoalveolar lavage specimens consistently show increased numbers of CD4+ and CD8+ T lymphocytes. 2. Most affected patients have specific antibodies against the offending antigen in their serum. 3. Complement and immunoglobulins have been demonstrated within vessel walls by immunofluorescence. 4. Noncaseating granulomas are found in the lungs of two-thirds of affected patients.

Cytomegalovirus (CMV) Infection: Manifestations, and High-Risk Populations (4)

1. CMV is a member of the herpesvirus family. 2. Infected cells exhibit gigantism of both cytoplasm and nucleus, with large nuclear inclusions and clear halos ("owl's eye"). 3. Classic cytomegalic inclusion disease occurs in neonates. 4. CMV pneumonitis is a serious problem in immunosuppressed adults, such as patients with AIDS and recipients of allogeneic hematopoietic stem cell transplants.

Clinical Features of Coal Worker's Pneumoconiosis (CWP) and Progressive Massive Fibrosis (PMF) (5)

1. CWP is usually a benign disease with little impact on lung function. 2. PMF development leads to increased pulmonary dysfunction, pulmonary hypertension, and cor pulmonale. 3. Progression from CWP to PMF is linked to higher coal dust exposure levels and total dust burden. 4. PMF tends to progress even in the absence of further exposure. 5. No increased frequency of lung carcinoma in coal miners, distinguishing CWP from silica and asbestos exposures.

Morphology of Candida albicans (4)

1. Candida albicans demonstrates yeast-like forms (blastoconidia), pseudohyphae, and true hyphae in tissue sections. 2. Pseudohyphae are an important diagnostic clue, representing budding yeast cells joined end to end at constrictions, simulating true fungal hyphae. 3. The organisms may be visible with routine hematoxylin and eosin (H&E) stains. 4. Special "fungal" stains such as Gomori methenamine-silver and periodic acid-Schiff are commonly used to better highlight the pathogens.

Opportunistic Fungal Infections: Candidiasis: Candida Albicans, Overview (3)

1. Candida albicans is the most common disease-causing fungus 2. It is a normal inhabitant of the oral cavity, gastrointestinal tract, and vagina in many individuals 3. Systemic candidiasis (with associated pneumonia) is a disease restricted to immunocompromised patients that has protean manifestations.

Carcinoid Tumors: Characteristics, Classification, and Epidemiology

1. Carcinoid tumors are malignant tumors composed of cells containing dense-core neurosecretory granules in their cytoplasm and may occasionally secrete hormonally active polypeptides. 2. They are considered low-grade neuroendocrine carcinomas. 3. Carcinoid tumors are subclassified as typical or atypical; both types are often resectable and curable. 4. They occasionally occur as part of the multiple endocrine neoplasia syndrome. 5. Bronchial carcinoids occur in young adults (average age 40 years) and represent about 5% of all pulmonary neoplasms.

Morphology and Histological Features of Carcinoid Tumors in the Lung ()

1. Carcinoid tumors mostly originate in main bronchi and grow in two patterns: (a) an obstructing polypoid, spherical, intraluminal mass (b) a mucosal plaque penetrating the bronchial wall to fan out in the peribronchial tissue, known as the collar-button lesion. 2. Peripheral carcinoids are less common. 3. 5% to 15% of carcinoids have metastasized to the hilar nodes at presentation, but distant metastases are rare. 4. Typical carcinoids are composed of nests of uniform cells with regular round nuclei with "salt-and-pepper" chromatin, absent or rare mitoses and little pleomorphism. 5. Atypical carcinoid tumors display a higher mitotic rate and small foci of necrosis, have a higher incidence of lymph node and distant metastasis than typical carcinoids and contain TP53 mutations in 20% to 40% of cases. 6. Typical carcinoid, atypical carcinoid, and large cell neuroendocrine and small cell carcinoma can be viewed as a continuum of increasing histologic aggressiveness and malignant potential within the spectrum of pulmonary neuroendocrine neoplasms.

Small Cell Lung Carcinomas (SCLCs): Features, Morphology, Characteristics and Mixed patterns (9)

1. SCLCs generally appear as pale gray, centrally located masses that extend into the lung parenchyma. 2. Composed of relatively small tumor cells with a round to fusiform shape, scant cytoplasm, and finely granular chromatin with a salt and pepper appearance. 3. Numerous mitotic figures are present, and necrosis is invariably present and may be extensive. 4. The tumor cells are fragile and often show fragmentation and "crush artifact" in small biopsy specimens. 5. Another feature is nuclear molding resulting from close apposition of tumor cells that have scant cytoplasm. 6. These tumors express a variety of neuroendocrine markers and may secrete a host of polypeptide hormones that may result in paraneoplastic syndromes. 7. By the time of diagnosis, most will have metastasized to hilar and mediastinal lymph nodes. 8. SCLC is grouped together with large cell neuroendocrine carcinoma, another very aggressive tumor that exhibits neuroendocrine morphology and expresses neuroendocrine markers. 9. Mixed patterns (e.g., adenosquamous carcinoma, mixed adenocarcinoma, small cell carcinoma) are seen in 10% or less of lung carcinomas.

Sarcoidosis: Etiology, Pathogenesis, and Immunologic Abnormalities (8)

1. Sarcoidosis etiology remains unknown, but it's suggested to be a disease of disordered immune regulation in genetically predisposed individuals exposed to certain environmental agents. 2. Immunologic abnormalities in sarcoidosis suggest the development of a cell-mediated response to an unidentified antigen, driven by CD4+ helper T cells. 3. These immunologic clues include: a. Intraalveolar and interstitial accumulation of CD4+ TH1 cells, with peripheral T cell cytopenia. b. Oligoclonal expansion of CD4+ TH1 T cells within the lung. c. Increases in TH1 cytokines such as IL-2 and IFN-γ, resulting in T cell proliferation and macrophage activation. d. Increases in several cytokines that favor recruitment of additional T cells and monocytes, and contribute to granuloma formation. e. Anergy to common skin test antigens. f. Polyclonal hypergammaglobulinemia. 4. Familial and racial clustering of cases suggest the involvement of genetic factors. 5. Sarcoidosis recurs in the new lungs after lung transplantation in at least one-third of patients, but without any effect on survival. 6. Several putative antigens have been proposed as the inciting agent for sarcoidosis, but no specific antigen or infectious agent has been definitively linked to the disease.

Sarcoidosis: Characteristics, Manifestations, and Epidemiology (5)

1. Sarcoidosis is a multisystem disease of unknown etiology characterized by noncaseating granulomatous inflammation in various tissues and organs. 2. Presentation can include restrictive lung disease, bilateral hilar lymphadenopathy, or lung involvement, and is visible on chest radiographs in most cases. 3. Eye and skin involvement each occurs in about 25% of cases and may occasionally be the presenting feature. 4. Sarcoidosis affects both genders, all races, and age groups but has some interesting epidemiologic trends: a. Predilection for adults younger than 40 years. b. High incidence in Danish and Swedish populations and African Americans in the United States. c. Higher prevalence among nonsmokers. 5. Histologic diagnosis of sarcoidosis is one of exclusion due to similarities with other diseases that produce noncaseating granulomas.

Staphylococcus aureus: Association with Respiratory Infections and Complications (4)

1. Secondary Bacterial Pneumonia: - S. aureus is an important cause of secondary bacterial pneumonia in children and healthy adults after viral respiratory illnesses. 2. Complications: - Staphylococcal pneumonia is associated with a high incidence of complications, such as lung abscess and empyema. 3. Intravenous Drug Abuse: - Staphylococcal pneumonia occurring in association with right-sided staphylococcal endocarditis is a serious complication of intravenous drug abuse. 4. Nosocomial Pneumonia: - S. aureus is also an important cause of nosocomial pneumonia.

Lung Carcinomas: Epidemiology, Histologic Types, and Treatment Approaches (4)

1. Carcinoma of the lung is the leading cause of cancer-related deaths in industrialized (מתועשת) countries. - The peak incidence is in individuals in their fifties and sixties. - At diagnosis, over 50% of patients already have distant metastases, and an additional 25% have disease in the regional lymph nodes. - The 5-year survival rate is about 16% overall, and 45% for disease localized to the lung. 2. The four major histologic types of lung carcinomas are: (1) Adenocarcinoma (2) Squamous cell carcinoma (3) Small cell carcinoma - (a subtype of neuroendocrine carcinoma) (4) Large cell carcinoma. - Squamous cell and small cell carcinomas have the strongest association with smoking. - Adenocarcinoma has become the most common primary lung tumor, especially in women, never-smokers, and individuals younger than 45 years. 3. Lung carcinomas were previously classified into two broad groups: small cell lung cancer (SCLC) and non-small cell lung cancer (NSCLC). - SCLC is usually metastatic at diagnosis and best treated with systemic chemotherapy, with or without radiation therapy. - NSCLC is more likely to be resectable and usually responds poorly to conventional chemotherapy. However, targeted therapies and immunotherapy approaches are now available for a subset of NSCLC.

Primary Lung Tumors: Composition and Characteristics of Most Common Benign Tumor (4)

1. Carcinomas make up roughly 95% of primary lung tumors. 2. The remaining 5% are a miscellaneous group that includes carcinoids, mesenchymal malignancies (e.g., fibrosarcomas, leiomyomas), lymphomas, and a few benign lesions. 3. The most common benign tumor is often referred to as a "Hamartoma." - It is spherical and small (1 to 4 cm). - Often shows up as a "coin lesion" on chest imaging. - Mainly consists of mature cartilage, fat, fibrous tissue, and blood vessels. - Clonal cytogenetic abnormalities have been identified, demonstrating that it is a benign neoplasm rather than a developmental anomaly, thus "hamartoma" is a misnomer.

Opportunistic Fungal Infections: Cryptococcosis: C. neoformans, Overview (3)

1. Caused by C. neoformans 2. Manifests as an opportunistic infection in immunocompromised hosts 3. Particularly patients with AIDS or hematolymphoid malignancies.

Nontuberculous Mycobacterial Disease: Manifestations, Associated Conditions, and Histopathology (3)

1. Causes chronic, localized pulmonary disease in immunocompetent individuals, often mimicking tuberculosis, especially in patients with a history of smoking or alcohol abuse. 2. Common strains in the United States include Mycobacterium avium-intracellulare (M. avium complex), Mycobacterium kansasii, and Mycobacterium abscessus. 3. In immunosuppressed individuals, such as HIV-seropositive patients, M. avium complex infection manifests as disseminated disease with systemic signs and symptoms, hepatosplenomegaly, lymphadenopathy, gastrointestinal symptoms, and pulmonary involvement similar to tuberculosis.

Aspiration Pneumonia: Causes, Characteristics, Complications, and Microaspiration (4)

1. Causes: (1) Debilitated patients (2) Aspirating gastric contents while unconscious (3) Repeated vomiting. 2. Characteristics: Abnormal gag and swallowing reflexes, chemical and bacterial components, and often necrotizing. 3. Complications: Rapid disease progression often causes death in vulnerable people, while survivors may develop abscesses. 4. Microaspiration: Occurs in many individuals, especially those with gastroesophageal reflux, and may exacerbate other lung diseases but does not lead to pneumonia.

Morphology of CMV-Infected Cells: Histological Features and Affected Cell Types (4)

1. Characteristic enlargement of CMV-infected cells is evident in histological examination. 2. Affected cell types vary by organ: parenchymal epithelial cells in glandular organs, neurons in the brain, alveolar macrophages and epithelial and endothelial cells in the lungs, and tubular epithelial and glomerular endothelial cells in the kidneys. 3. Infected cells are enlarged, often up to 40 µm in diameter, and exhibit cellular and nuclear pleiomorphism. 4. Prominent intranuclear basophilic inclusions, often surrounded by a clear halo, and smaller basophilic inclusions in the cytoplasm can be seen.

Secondary Tuberculosis: Definition, Localization, and Relationship with HIV Infection (5)

1. Secondary tuberculosis arises in a previously sensitized host. 2. May appear after primary tuberculosis, reactivation of dormant primary lesions, or reinfection. - More commonly arises from reactivation of dormant primary lesions many decades after initial infection, particularly when host resistance is weakened 3. Classically localized to the apex of one or both upper lobes. - As a result of this localization, the regional lymph nodes are less prominently involved early in the disease than they are in primary tuberculosis. 4. Cavitation occurs readily in secondary tuberculosis, leading to erosion into and dissemination along airways. - Patient now produces sputum containing bacilli. 5. In HIV-positive patients, the manifestations of secondary tuberculosis differ depending on the degree of immunosuppression.

Silicosis: Clinical Features, Complications, and Associations (7)

1. Silicosis is often detected on routine chest radiographs in asymptomatic workers, showing fine nodularity in the upper lung zones. 2. Shortness of breath usually develops late in the course, after PMF is present. 3. Patients with PMF may develop pulmonary hypertension and cor pulmonale due to chronic hypoxia-induced vasoconstriction and parenchymal destruction. 4. The disease is slowly progressive and can severely limit physical activity. 5. Silicosis is associated with increased susceptibility to tuberculosis, possibly due to depressed cell-mediated immunity and impaired macrophage function. 6. Nodules of silicotuberculosis often contain a central zone of caseation. 7. The relationship between silica and lung cancer is unsettled, but most studies suggest an increased risk with silica exposure.

Silicosis: Prevalence, Causes, and Pathophysiology (6)

1. Silicosis is the most prevalent chronic occupational disease globally. 2. Caused by inhalation of crystalline silica, mostly in occupational settings. 3. High-risk workers include sandblasters and hard-rock miners. 4. Crystalline forms of silica (quartz, cristobalite, tridymite) are the most toxic and fibrogenic. 5. Ingested silica particles activate the inflammasome and release inflammatory mediators. 6. Fibrogenic effect of quartz is reduced when mixed with other minerals, such as hematite.

Pathogenesis of Pulmonary Hypertension: Common Causes and Genetic Basis of "Idiopathic" Pulmonary Arterial Hypertension (6)

1. Chronic obstructive or interstitial lung diseases (group 3): - Obliterate alveolar capillaries, increasing pulmonary resistance and blood pressure 2. Antecedent congenital or acquired heart disease (group 2): - Increase in left atrial pressure and pulmonary venous pressure leading to hypertension 3. Recurrent thromboemboli (group 4): - Reduce functional cross-sectional area of the pulmonary vascular bed, increasing vascular resistance 4. Autoimmune diseases (group 1): - Involve pulmonary vasculature and/or interstitium, leading to increased vascular resistance and pulmonary hypertension 5. Obstructive sleep apnea (group 3): - Associated with obesity and hypoxemia, contributes to pulmonary hypertension and cor pulmonale. 6. "Idiopathic" pulmonary arterial hypertension: - Up to 80% have a genetic basis, with BMPR2 mutations being most common - Modifier genes and/or environmental triggers may contribute to the disorder

Pathogenesis of Pneumoconioses: Variables, Particle Sizes, Macrophages, and Smoking Effects (4)

1. Size, shape, solubility, and reactivity of particles, all affecting lung reaction to mineral dusts: 2. 1-5 μm in diameter most dangerous, lodged at distal airway bifurcations 3. The pulmonary alveolar macrophage is a key cellular element in the initiation and perpetuation of infammation, lung injury and fbrosis. 4. Tobacco smoking: Worsens the effects of all inhaled mineral dusts, especially asbestos

Clinical Features, Diagnosis, and Prognosis of Tuberculosis (6)

1. Clinical features: Insidious onset, systemic and localizing symptoms and signs, including malaise, anorexia, weight loss, fever, night sweats, increasing amounts of sputum, hemoptysis, and pleuritic pain. - Commonly, the fever is low grade and remittent (appearing late each afternoon and then subsiding) - Systemic manifestations, probably related to the release of cytokines by activated macrophages (e.g., TNF and IL-1) 2. Extrapulmonary manifestations depend on the organ system involved - (e.g., tuberculous salpingitis may present as infertility, tuberculous meningitis with headache and neurologic deficits, Pott disease with back pain and paraplegia) 3. When cavitation is present, the sputum contains tubercle bacilli. 4. Diagnosis: Identification of tubercle bacilli, with the most common method being demonstration of acid-fast organisms in sputum by staining or fluorescent auramine rhodamine. 5. Multidrug resistance (MDR) is a growing concern, with increasing rates of resistance to two or more primary drugs used for treatment of tuberculosis. 6. Prognosis: Determined by the extent of infection, immune status of the host, and antibiotic sensitivity of the organism; guarded prognosis in multidrug-resistant tuberculosis.

Asbestosis: Clinical Features, Complications, and Risks (4)

1. Clinical findings in asbestosis are similar to other chronic interstitial lung diseases. 2. Symptoms include progressively worsening dyspnea, cough, and sputum production, appearing 10 to 20 years after exposure. 3. The disease may progress to congestive heart failure, cor pulmonale, and death. 4. Asbestos exposure increases the risk for lung carcinoma and malignant mesothelioma, with smoking further increasing lung carcinoma risk.

Clinical Features and Consequences of Pulmonary Thromboembolism: Silent Emboli, Severe Consequences, Obstruction to Small-Medium Pulmonary Branches, Recurrent Emboli (4)

1. Clinically silent emboli (60-80%): - Small emboli that do not cause symptoms. - Bronchial circulation sustains lung parenchyma viability, and embolic mass is removed by fibrinolytic activity. 2. Severe consequences (5%): - Death, acute right-sided heart failure, or cardiovascular collapse. - Occurs when more than 60% of total pulmonary vasculature is obstructed by large or multiple small emboli. 3. Obstruction of small to medium pulmonary branches (10-15%): - Causes pulmonary infarction with circulatory insufficiency. - Presents with dyspnea. 4. Recurrent emboli (<3%): - Leads to pulmonary hypertension, chronic right-sided heart failure, and pulmonary vascular sclerosis. - Causes progressively worsening dyspnea.

Community-Acquired Viral Pneumonias: Causes, Pathology, Radiologic Appearance, and Risk Factors (4)

1. Common causes: Influenza types A and B Respiratory syncytial viruses Human metapneumovirus Adenovirus Rhinoviruses Rubeola virus Varicella virus 2. Pathology: Infection and damage to respiratory epithelium Inflammatory response Interstitial inflammation when alveoli are involved Outpouring of fluid into alveolar spaces may occur 3. Radiologic appearance: Changes may mimic those of bacterial pneumonia Cannot distinguish bacterial and viral pneumonia based on radiologic appearance alone 4. Risk factors for serious complications: Infants Older adults Malnourished patients Alcoholics Immunosuppressed individuals

Goodpasture Syndrome: Demographics, Risk Factors, and Treatment Options (4)

1. Commonly affects patients in their teens or twenties 2. Male predominance 3. Majority are active smokers 4. Treatment options: Plasmapheresis Immunosuppressive therapy Renal transplantation for severe renal disease

The Pneumonia Syndromes and Implicated Pathogens (7)

1. Community-Acquired Bacterial Pneumonia: - Streptococcus pneumoniae - Haemophilus influenzae - Moraxella catarrhalis - Staphylococcus aureus 2. Community-Acquired Viral Pneumonia: - Respiratory syncytial virus - Human metapneumovirus - Parainfluenza virus (children) - Influenza A and B (adults) - Adenovirus (military recruits) 3. Nosocomial Pneumonia: - Gram-negative rods belonging to Enterobacteriaceae (Klebsiella spp., Serratia marcescens, Escherichia coli) - Pseudomonas spp. 4. Aspiration Pneumonia: - Anaerobic oral flora (Bacteroides, Prevotella, Fusobacterium, Peptostreptococcus) 5. Chronic Pneumonia: - Nocardia - Actinomyces - Granulomatous 6. Necrotizing Pneumonia and Lung Abscess: - Anaerobic bacteria (extremely common) 7. Pneumonia in the Immunocompromised Host: - Cytomegalovirus - Pneumocystis jiroveci - Mycobacterium avium complex (MAC)

Etiology and Pathogenesis of Lung Cancer: Smoking-related Carcinomas, Adenocarcinomas, Carcinogenic influences, Squamous Cell Carcinomas (5)

1. Smoking-related carcinomas: - Arise from the accumulation of driver mutations in benign progenitor cells in the lung. - Inactivation of tumor suppressor genes on chromosome 3 (3p) is a common early event. - TP53 tumor suppressor gene and KRAS oncogene mutations occur relatively late. - Genetic changes, such as loss of 3p chromosomal material, are found even in benign bronchial epithelium of smokers without lung cancer. 2. Adenocarcinomas: - A subset (10% in whites, 30% in Asians), particularly in nonsmoking women, harbor mutations activating the epidermal growth factor receptor (EGFR), making them sensitive to drugs that inhibit EGFR signaling. - EGFR and KRAS mutations are mutually exclusive as KRAS lies downstream of EGFR. - Other targetable mutations activate other tyrosine kinases (e.g., ALK, ROS1, HER2, c-MET) and are present in a low frequency of adenocarcinomas (4-6%). 3. Carcinogenic influences: - About 90% of lung cancers occur in active smokers or recent quitters. - There is a nearly linear correlation between the frequency of lung cancer and pack-years of cigarette smoking. - The increased risk is 60 times greater among habitual heavy smokers than among nonsmokers. - The risk never returns to baseline levels even after smoking cessation. - Passive smoking, smoking of pipes and cigars, and certain occupational exposures (e.g., work in uranium mines, work with asbestos, and inhalation of dusts containing arsenic, chromium, nickel, or vinyl chloride) also increase the risk. - Only about 11% of heavy smokers develop lung cancer, indicating the likely influence of genetic factors. 4. Adenoma-carcinoma sequence: - Some invasive adenocarcinomas of the lung arise through an atypical adenomatous hyperplasia-adenocarcinoma in situ-invasive adenocarcinoma sequence. - Bronchioalveolar stem cells (BASCs) at the bronchioloalveolar duct junction proliferate and replenish the normal cell types after lung injury, potentially leading to pulmonary adenocarcinomas upon acquiring a somatic KRAS mutation. 5. Squamous cell carcinomas: - Development correlates linearly with the intensity of exposure to cigarette smoke. - Epithelial changes begin with basal cell hyperplasia and squamous metaplasia, progressing to squamous dysplasia and carcinoma in situ, before culminating in invasive cancer.

Squamous Cell Carcinomas of the Lung: Characteristics, Progression, and Histology (7)

1. Squamous cell carcinomas of the lung are more common in men than women and are closely correlated with a smoking history. 2. These carcinomas tend to arise centrally in major bronchi, spreading to local hilar nodes, but disseminate outside the thorax later than other histologic types. 3. Large lesions may undergo central necrosis, leading to Cavitation. 4. Squamous cell carcinomas are often preceded by years of squamous metaplasia or dysplasia in the bronchial epithelium, transforming to carcinoma in situ, a phase that may last for several years. 5. Atypical cells may be identified in cytologic smears of sputum or in bronchial lavage fluids or brushings, although the lesion is asymptomatic and undetectable on radiographs. 6. Eventually, the neoplasm reaches a symptomatic stage, when a well-defined tumor mass begins to obstruct the lumen of a major bronchus, often producing distal atelectasis and infection. 7. On histologic examination, these tumors range from well-differentiated squamous cell neoplasms showing keratin pearls and intercellular bridges to poorly differentiated neoplasms exhibiting only minimal squamous cell features.

Extrapulmonary Tuberculosis Morphology: Forms and Manifestations (5)

1. Systemic miliary tuberculosis: - Formed when the organisms disseminate hematogenously throughout the body - Most prominent in the liver, bone marrow, spleen, adrenal glands, meninges, kidneys, fallopian tubes, and epididymis 2. Isolated-organ tuberculosis: - May affect meninges (tuberculous meningitis), kidneys (renal tuberculosis), adrenal glands, bones (osteomyelitis), and fallopian tubes (salpingitis). - May appear in any one of the organs or tissues seeded hematogenously and may be the presenting manifestation of tuberculosis 3. Pott disease: - Tuberculosis affecting the vertebrae. - Paraspinal "cold" abscesses containing necrotic tissues may track along the tissue planes to present as an abdominal or pelvic mass. 4. Lymphadenitis: - Most frequent form of extrapulmonary tuberculosis, usually occurs in the cervical region ("scrofula"). 5. Intestinal tuberculosis: - Rare in developed countries, occurs due to swallowing coughed up infective material, and affects mucosal lymphoid aggregates of the small and large bowel.

Tuberculosis (TB): Overview, Epidemiology, Transmission, Infection vs Disease, and Tuberculin Test (5)

1. TB is a communicable chronic granulomatous disease caused by Mycobacterium tuberculosis, usually involving the lungs but can affect any organ or tissue. 2. It is the most common cause of death resulting from a single infectious agent, with 1.7 billion individuals infected worldwide. 3. TB is more prevalent in conditions of poverty, crowding, and chronic debilitating illness, and has higher rates among certain groups and individuals with specific disease states. - Although other routes may be involved, most infections are acquired by direct person-to-person transmission of airborne droplets of organisms from an active case to a susceptible host 4. Infection implies seeding of a focus with organisms, while disease involves clinically significant tissue damage. Infection can reactivate to produce communicable and life-threatening disease when immune defenses are lowered. 5. Infection with M. tuberculosis typically leads to the development of delayed hypersensitivity which can be detected by The tuberculin (Mantoux) test detects delayed hypersensitivity to tubercular antigens but does not differentiate between infection and disease. False-negative and false-positive reactions are possible.

Pneumonia in the Immunocompromised Host: Common Opportunistic Pathogens (3)

1. The appearance of a pulmonary infiltrate and signs of infection (e.g., fever) are some of the most common and serious complications in individuals with immune systems that are suppressed by disease, immunosuppressive drugs, or therapeutic irradiation 2. The more common pulmonary pathogens include (1) bacteria (P. aeruginosa, Mycobacterium spp., L. pneumophila, and Listeria monocytogenes) (2) viruses (cytomegalovirus and herpesvirus) (3) fungi (P. jiroveci, Candida spp., Aspergillus spp., and Cryptococcus neoformans)

Alveolar walls: 3 Components

1. The capillary endothelium and basement membrane. 2. The pulmonary interstitium composed of fine elastic fibers, small bundles of collagen, a few fibroblast-like cells, smooth muscle cells, mast cells, and rare mononuclear cells. 3. Alveolar epithelium

Sarcoidosis: Morphology, Lung Involvement, and Lymph Node Characteristics

1. The cardinal histopathologic feature of sarcoidosis is the nonnecrotizing epithelioid granuloma (figure), a compact collection of epithelioid cells rimmed by an outer zone rich in CD4+ T cells. 2. Two other microscopic features sometimes seen in the granulomas are Schaumann bodies (laminated concretions composed of calcium and proteins) and Asteroid bodies (stellate inclusions enclosed within giant cells). 3. Lungs are involved in 90% of patients, with granulomas predominantly involving the interstitium rather than air spaces and a tendency to localize around bronchioles, pulmonary venules, and in the pleura. 4. In 5% to 15% of patients, the granulomas are replaced by diffuse interstitial fibrosis, resulting in a "honeycomb lung." 5. Intrathoracic hilar and paratracheal lymph nodes are enlarged in 75% to 90% of patients, while one-third present with peripheral lymphadenopathy. 6. Lymph nodes in sarcoidosis are characteristically painless, firm, rubbery, nonmatted (nonadherent), and do not ulcerate.

Goodpasture Syndrome: Autoimmune Disease, Target, and Effects on Lung and Kidney (3)

1. Uncommon autoimmune disease 2. Involves circulating autoantibodies against specific domains of type IV collagen - Intrinsic to the basement membranes of renal glomeruli and pulmonary alveoli 3. Effects: - Necrotizing hemorrhagic interstitial pneumonitis (lung) - Rapidly progressive glomerulonephritis (kidney)

Large Cell Carcinoma of the Lung: Characteristics and Histology (3)

1. Undifferentiated malignant epithelial tumors that lack the cytologic features of neuroendocrine carcinoma 2. Show no evidence of glandular or squamous differentiation. 3. The cells typically have large nuclei, prominent nucleoli, and moderate amounts of cytoplasm.

Nonmalignant Lesions of the Larynx: Vocal Cord Nodules and Laryngeal Papilloma (2)

1. Vocal cord nodules: - Smooth, hemispherical protrusions, usually <0.5 cm in diameter, located on the true vocal cords. - Composed of fibrous tissue and covered by intact stratified squamous mucosa. - Associated with chronic irritation or overuse, often seen in heavy smokers or singers (singer's nodes). 2. Laryngeal papilloma (squamous papilloma of the larynx): - Benign neoplasm, typically located on the true vocal cords. - Forms soft, raspberry-like excrescence, rarely exceeding 1 cm in diameter. - Histologically characterized by slender, fingerlike projections with central fibrovascular cores and covered by stratified squamous epithelium. - Can lead to ulceration and hemoptysis if trauma occurs. - Typically single in adults but multiple in children with recurrent respiratory papillomatosis (RRP) caused by human papillomavirus (HPV) types 6 and 11. - Spontaneous regression often occurs at puberty, and cancerous transformation is rare.

Which of the following statements about Goodpasture Syndrome is NOT true? A) Affects mainly patients in their 40s or 50s B) Has a male predominance C) Majority of patients are active smokers D) Plasmapheresis is a treatment option E) Renal transplantation may be required for severe renal disease

A) Affects mainly patients in their 40s or 50s. Goodpasture Syndrome typically occurs in patients in their teens or twenties, has a male predominance, and is more common in active smokers. Treatment options include plasmapheresis, immunosuppressive therapy, and renal transplantation for severe renal disease.

What is the main difference between antigenic drift and antigenic shift in the context of influenza infections? A) Antigenic drift involves small mutations, while antigenic shift involves recombination of RNA segments. B) Antigenic drift causes pandemics, while antigenic shift causes epidemics. C) Antigenic drift is restricted to humans, while antigenic shift affects animals as well. D) Antigenic drift is related to the lipid bilayer, while antigenic shift is related to the nucleoprotein. E) None of the above.

A) Antigenic drift involves small mutations, while antigenic shift involves recombination of RNA segments. Antigenic drift occurs through mutations of the hemagglutinin and neuraminidase antigens that allow the virus to escape most host antibodies, causing epidemics. Pandemics, which last longer and are more widespread than epidemics, may occur when both the hemagglutinin and neuraminidase are replaced through recombination of RNA segments with those of animal viruses, making all animals susceptible to the new influenza virus (antigenic shift).

What is the most common genetic mutation associated with "idiopathic" pulmonary arterial hypertension? A) BMPR2 B) PTPN11 C) KCNJ11 D) SCN1A E) CFTR

A) BMPR2. Inactivating germ line mutations in the gene encoding bone morphogenetic protein receptor 2 (BMPR2) are found in 75% of familial cases of pulmonary hypertension and 25% of sporadic cases. These mutations are believed to lead to dysfunction of endothelium and proliferation of vascular smooth muscle cells in the pulmonary vasculature.

A 65-year-old man with a history of asbestos exposure presents with dyspnea and chest pain. CT scan reveals extensive pleural fibrosis and plaque formation. Which of the following is most likely the morphological appearance of the man's tumor on biopsy? A) Cuboidal cells with small papillary buds lining tubular and microcystic spaces B) Spindled, occasionally fibroblastic-appearing cells growing in sheets C) Both cuboidal cells lining spaces and spindled cells growing in sheets D) Large anaplastic cells with pleomorphism E) Clear cells forming gland-like structures

A) Cuboidal cells with small papillary buds lining tubular and microcystic spaces The patient's history of asbestos exposure, dyspnea, and chest pain, coupled with the CT findings, suggests malignant mesothelioma. The most common morphological appearance of malignant mesothelioma is the epithelial type, which is characterized by cuboidal cells with small papillary buds lining tubular and microcystic spaces. This type can often be confused with a pulmonary adenocarcinoma.

A 50-year-old male presents with persistent hoarseness for the past three months. The patient has a long history of smoking and occasional alcohol consumption. Laryngoscopy reveals pearly gray, wrinkled plaques on the vocal cords. What is the most likely diagnosis? A) Glottic squamous cell carcinoma B) Supraglottic squamous cell carcinoma C) Subglottic squamous cell carcinoma D) Laryngeal adenocarcinoma E) Neuroendocrine carcinoma of the larynx

A) Glottic squamous cell carcinoma. The patient's clinical presentation of persistent hoarseness, along with the presence of pearly gray, wrinkled plaques on the vocal cords, is consistent with glottic squamous cell carcinoma. Glottic tumors have a better prognosis compared to supraglottic or subglottic tumors due to early symptom development and less spread beyond the larynx.

A 23-year-old male presents with a history of recurrent episodes of hemoptysis and worsening shortness of breath. Physical examination reveals bibasilar crackles on lung auscultation. A chest CT scan shows patchy areas of consolidation in both lungs. A lung biopsy is performed, and microscopic examination reveals focal necrosis of alveolar walls, intra-alveolar hemorrhage, fibrous thickening of septa, and hypertrophic type II pneumocytes. Immunofluorescence staining shows linear immunoglobulin deposition along the alveolar septa. Based on the biopsy findings, what is the most likely diagnosis? A) Goodpasture syndrome B) Systemic lupus erythematosus C) Wegener's granulomatosis D) Sarcoidosis E) Idiopathic pulmonary fibrosis

A) Goodpasture syndrome. The lung biopsy findings of focal necrosis of alveolar walls, intra-alveolar hemorrhage, fibrous thickening of septa, hypertrophic type II pneumocytes, and linear immunoglobulin deposition along the alveolar septa are consistent with Goodpasture syndrome, an autoimmune disease that targets specific domains of type IV collagen and affects both the lungs and kidneys.

Which of the following fibrosing diseases has a better prognosis than idiopathic pulmonary fibrosis (IPF)? A) Nonspecific interstitial pneumonia (NSIP) B) Cryptogenic organizing pneumonia C) Systemic sclerosis D) Rheumatoid arthritis E) Systemic lupus erythematosus

A) Nonspecific interstitial pneumonia (NSIP). NSIP has a better prognosis than IPF and is characterized by mild to moderate interstitial chronic inflammation and/or fibrosis, which is patchy but uniform in the areas involved. NSIP is also frequently associated with collagen vascular disorders, such as rheumatoid arthritis.

Which of the following presentations of Candidiasis is most common? A) Oral thrush B) Vaginitis C) Esophagitis D) Skin infection E) Invasive candidiasis

A) Oral thrush. It is the most common presentation of Candidiasis and is characterized by gray-white, dirty-looking pseudomembranes on the oral mucosa. It is associated with newborns, debilitated patients, oral corticosteroids, broad-spectrum antibiotics, and HIV-positive patients.

A 58-year-old male smoker presents with a persistent cough, weight loss, and left supraclavicular swelling. Further examination reveals ptosis, miosis, and anhidrosis on the right side of his face. Imaging studies show an apical mass in the right lung and destruction of the first and second ribs. Based on the patient's symptoms and imaging findings, what is the most likely diagnosis? A) Pancoast tumor B) Mediastinal mass C) Mesothelioma D) Metastatic carcinoma from another primary site E) Lymphoma

A) Pancoast tumor. The patient's symptoms of ptosis, miosis, and anhidrosis (Horner's syndrome) in conjunction with the imaging findings are indicative of a Pancoast tumor.

Which of the following is NOT a characteristic feature of silicotic nodules? A) Primarily found in the lower lung zones B) Microscopically show concentric hyalinized collagen fibers surrounding an amorphous center C) Distinctive "whorled" appearance of collagen fibers D) Weakly birefringent silica particles revealed by polarized microscopy E) May coalesce into hard, collagenous scars and progress to PMF

A) Primarily found in the lower lung zones. Silicotic nodules are predominantly found in the upper lung zones, not the lower lung zones.

A 60-year-old patient with a history of congestive heart failure presents with shortness of breath. A pleural effusion is discovered on chest imaging. Which of the following is most likely to characterize the pleural effusion in this patient? A) Transudate with low protein content B) Exudate with high protein content and inflammatory cells C) Exudate caused by microbial invasion D) Exudate associated with systemic lupus erythematosus E) Hemorrhagic effusion with malignant cells

A) Transudate with low protein content This patient's history of congestive heart failure, a common cause of bilateral hydrothorax, suggests that the pleural effusion is likely a transudate, characterized by lower protein content. Exudates, on the other hand, are typically associated with conditions such as pleuritis, which is characterized by a high protein content and often the presence of inflammatory cells.

OBSTRUCTIVE LUNG (AIRWAY) DISEASES: Asthma Clinical features Therapy

An attack of asthma is characterized by severe dyspnea and wheezing due to bronchoconstriction and mucus pugging, which leads to trapping of air in distal airspaces and progressive hyperinflation of the lungs. Standard therapies include anti-inflammatory drugs, particularly glucocorticoids, and bronchodilators such as beta-adrenergic drugs and leukotriene inhibitors (recall that leukotrienes are potent bronchoconstrictors).

Asbestos Exposure and Related Diseases (6)

Asbestos is a family of crystalline hydrated silicates with a fibrous geometry. On the basis of epidemiologic studies, occupational exposure to asbestos is linked to 1. Parenchymal interstitial fibrosis (asbestosis) 2. Localized fibrous plaques or, rarely, diffuse fibrosis in the pleura 3. Pleural effusions 4. Lung carcinoma 5. Malignant pleural and peritoneal mesothelioma 6. Laryngeal carcinoma

OBSTRUCTIVE LUNG (AIRWAY) DISEASES: Asthma Drug-Induced Asthma

Aspirin Patients with aspirin sensitivity present with recurrent rhinitis, nasal polyps, urticaria, and bronchospasm.

OBSTRUCTIVE LUNG (AIRWAY) DISEASES: Asthma Definition

Asthma is a chronic inflammatory disorder of the airways that causes recurrent episodes of wheezing, breathlessness, chest tightness, and cough, particularly at night and/or early in the morning.

OBSTRUCTIVE LUNG (AIRWAY) DISEASES: Asthma Pathogenesis

Asthma may be subclassified as atopic (evidence of allergen sensitization) or nonatopic. In both types, episodes of bronchospasm may be triggered by diverse exposures, such as respiratory infections (especially viral), airborne irritants (e.g., smoke, fumes), cold air, stress, and exercise.

ATELECTASIS (COLLAPSE) Treatment

Atelectasis (except when caused by contraction) is potentially reversible and should be treated promptly (בהקדם) to prevent hypoxemia and superimposed infection of the collapsed lung.

ATELECTASIS (COLLAPSE) Definition

Atelectasis, also known as collapse, is loss of lung volume caused by inadequate expansion of air spaces.

What is the primary site of damage in hypersensitivity pneumonitis? A) Bronchi B) Alveoli C) Pleura D) Lung vasculature E) None of the above

B) Alveoli. Hypersensitivity pneumonitis is an immunologically mediated inflammatory lung disease that primarily affects the alveoli and is often called allergic alveolitis. Unlike bronchial asthma, the damage in hypersensitivity pneumonitis occurs at the level of alveoli, manifesting predominantly as a restrictive lung disease with typical decreases in diffusion capacity, lung compliance, and total lung volume.

Which of the following is NOT a feature of Coal Worker's Pneumoconiosis (CWP)? A) Benign disease with little impact on lung function B) Associated with increased frequency of lung carcinoma C) Can progress to Progressive Massive Fibrosis (PMF) D) Usually occurs in coal miners

B) Associated with increased frequency of lung carcinoma. CWP is not associated with an increased frequency of lung carcinoma, which distinguishes it from both silica and asbestos exposures.

Who is most susceptible to Mycoplasma infections? A) Elderly individuals B) Children and young adults C) Pregnant women D) Immunocompromised individuals E) None of the above

B) Children and young adults are most susceptible to Mycoplasma infections

Which of the following is NOT refers to "Pulmonary Hemorrhage Syndromes"? A) Goodpasture syndrome B) Desquamative interstitial pneumonia C) Granulomatosis with polyangiitis D) Idiopathic pulmonary hemosiderosis E) All are correct

B) Desquamative interstitial pneumonia

In primary tuberculosis, what is the term for the combination of parenchymal and nodal lesions that occur when tubercle bacilli travel to regional lymph nodes? A) Ghon focus B) Ghon complex C) Ranke complex D) Caseating granuloma E) Noncaseating granuloma

B) Ghon complex. The Ghon complex is the combination of parenchymal and nodal lesions that occur when tubercle bacilli travel via lymphatic vessels to the regional lymph nodes, which also often caseate.

Which of the following statements best describes the appearance of a pulmonary infarct in its early stages? A) Pale and avascular B) Hemorrhagic and red-blue C) Yellow-brown with necrotic debris D) Gray-white with fibrous tissue E) Pink and edematous

B) Hemorrhagic and red-blue

Which paraneoplastic syndrome is most often associated with squamous cell neoplasms in patients with lung cancer? A) Syndrome of inappropriate secretion of anti-diuretic hormone (SIADH) B) Hypercalcemia C) Cushing syndrome D) Neuromuscular syndromes E) Coagulation abnormalities

B) Hypercalcemia. Hypercalcemia in lung cancer is often caused by the secretion of a parathyroid hormone-related peptide, and it is most frequently associated with squamous cell neoplasms. Although exceptions exist, the association of hypercalcemia with squamous cell neoplasms is a recognized pattern in the context of lung cancer.

What is a key diagnostic clue for Cryptococcus fungus identification? A) Presence of pseudohyphal forms B) Identification of the capsule C) Positive India ink staining D) Positive periodic acid-Schiff staining E) Negative cryptococcal latex agglutination assay

B) Identification of the capsule. The capsule of Cryptococcus fungus is a key diagnostic clue, as it is a thick, gelatinous structure surrounding the 5- to 10-μm yeast. India ink or periodic acid-Schiff staining effectively highlights the fungus, and cryptococcal latex agglutination assay is positive in more than 95% of patients infected with the organism.

A 58-year-old male with a history of progressive shortness of breath and dry cough for the past year undergoes a lung biopsy. The gross examination of the biopsy shows firm, rubbery white areas of fibrosis, preferentially within the lower lobe, subpleural regions, and along the interlobular septa. Histological examination reveals patchy interstitial fibrosis with varying intensity, and both early and late lesions are present. Dense fibrosis has caused the collapse of alveolar walls and formation of cystic spaces lined by hyperplastic type II pneumocytes. Based on the histological findings, what is the most likely diagnosis for this patient? A) Sarcoidosis B) Idiopathic Pulmonary Fibrosis (IPF) C) Chronic obstructive pulmonary disease (COPD) D) Bronchiectasis E) Tuberculosis

B) Idiopathic Pulmonary Fibrosis (IPF). The histological findings of patchy interstitial fibrosis with varying intensity, early and late lesions, and dense fibrosis causing collapse of alveolar walls and formation of cystic spaces lined by hyperplastic type II pneumocytes are characteristic of IPF.

Which of the following correctly describes the hyphae morphology of Rhizopus and Mucor? A) Septate, branching at acute angles B) Nonseptate, branching at right angles C) Septate, branching at right angles D) Nonseptate, branching at acute angles E) Aseptate, branching at obtuse angles

B) Nonseptate, branching at right angles. Mucormycosis is caused by the class of fungi known as Zygomycetes, such as Rhizopus and Mucor. Their hyphae are nonseptate and branch at right angles, while Aspergillus hyphae are septate and branch at more acute angles.

Which of the following is a typical histological finding in idiopathic pulmonary fibrosis (IPF)? A) Granulomas with multinucleated giant cells B) Patchy interstitial fibrosis with varying intensity C) Smooth muscle hypertrophy in bronchial walls D) Neutrophilic infiltration in alveolar spaces E) Formation of granulation tissue in the lung parenchyma

B) Patchy interstitial fibrosis with varying intensity. This is the hallmark of IPF, which involves exuberant fibroblastic proliferation and progresses to areas becoming more collagenous and less cellular over time.

What is the hallmark of chronic interstitial lung diseases? A) Hypoxia B) Reduced compliance (stiff lungs) C) Pulmonary hypertension D) Ground-glass shadows on chest radiographs E) Frequent overlap in histologic features

B) Reduced compliance (stiff lungs). The hallmark of chronic interstitial lung diseases is reduced compliance, which leads to increased effort to breathe (dyspnea).

A 45-year-old HIV-positive patient presents with a persistent cough, fever, and weight loss. Chest imaging shows consolidation in the lower and middle lobes, hilar lymphadenopathy, and noncavitary disease. Laboratory tests reveal a CD4+ T cell count of 180 cells/μL. What is the most likely diagnosis? A) Primary tuberculosis B) Secondary tuberculosis C) Pneumonia D) Lung cancer

B) Secondary tuberculosis with severe immunosuppression. The patient's clinical presentation, imaging findings, and low CD4+ T cell count suggest secondary tuberculosis with severe immunosuppression, which resembles progressive primary tuberculosis (lower and middle lobe consolidation, hilar lymphadenopathy, and noncavitary disease). In patients with severe immune deficiency, the frequency of extrapulmonary involvement also increases.

What is the most common histological type of laryngeal cancer? A) Adenocarcinoma B) Squamous cell carcinoma C) Neuroendocrine carcinoma D) Small cell carcinoma E) Undifferentiated carcinoma

B) Squamous cell carcinoma About 95% of laryngeal cancers are typical squamous cell carcinomas. Adenocarcinomas are rare and presumed to arise from mucous glands.

Community-Acquired Bacterial Pneumonias: Bacterial pneumonias, S. pneumoniae

Bacterial pneumonias often follow a viral upper-respiratory tract infection. S. pneumoniae (i.e., the pneumococcus) is the most common cause of community-acquired acute pneumonia

OBSTRUCTIVE LUNG (AIRWAY) DISEASES: Bronchiectasis - Main characteristics

Bronchiectasis gives rise to a characteristic symptom complex dominated by cough and expectoration (יְרִיקָה) of copious amounts of purulent sputum

OBSTRUCTIVE LUNG (AIRWAY) DISEASES: Bronchiectasis - Definition - Results from - Secondary disorder

Bronchiectasis is the permanent dilation of bronchi and bronchioles caused by destruction of smooth muscle and the supporting elastic tissue; it typically results from or is associated with chronic necrotizing infections. - It is not a primary disorder, as it always occurs secondary to persistent infection or obstruction caused by a variety of conditions.

Which of the following is NOT a disease linked to occupational exposure to asbestos? A) Parenchymal interstitial fibrosis (asbestosis) B) Pleural effusions C) Asthma D) Malignant pleural and peritoneal mesothelioma E) Laryngeal carcinoma

C) Asthma. Occupational exposure to asbestos is not linked to asthma. It is linked to asbestosis, pleural effusions, lung carcinoma, malignant pleural and peritoneal mesothelioma, and laryngeal carcinoma.

A 45-year-old former coal miner presents with shortness of breath and a dry cough. He worked in a coal mine for 20 years before retiring. A chest X-ray reveals multiple black scars in the upper lobes and upper zones of the lower lobes of his lungs. The scars measure between 2 cm and 4 cm in diameter. What is the most likely diagnosis for this patient? A) Pulmonary anthracosis B) Simple coal worker's pneumoconiosis (CWP) C) Complicated CWP or progressive massive fibrosis (PMF) D) Silicosis E) Asbestosis

C) Complicated CWP or progressive massive fibrosis (PMF). The patient's occupational history in coal mining and the presence of multiple dark black scars larger than 2 cm in diameter on the chest X-ray are consistent with complicated CWP or PMF.

A 68-year-old male with a known history of lung adenocarcinoma presents with sudden onset of chest pain and shortness of breath. On examination, the patient is tachycardic and hypotensive. Laboratory tests reveal prolonged prothrombin time (PT) and activated partial thromboplastin time (aPTT), elevated D-dimer, and decreased fibrinogen. Peripheral smear shows schistocytes. Based on the patient's history and laboratory findings, what is the most likely diagnosis? A) Syndrome of inappropriate secretion of anti-diuretic hormone (SIADH) B) Cushing syndrome C) Disseminated intravascular coagulation (DIC) D) Hypercalcemia E) Myasthenic syndrome

C) Disseminated intravascular coagulation (DIC)

A patient is diagnosed with nasopharyngeal carcinoma. Which of the following factors is most closely associated with this type of cancer? A) Human Papillomavirus infection B) Exposure to asbestos C) Epstein-Barr virus infection D) Excessive alcohol consumption E) Tobacco smoking

C) Epstein-Barr virus infection Epstein-Barr virus (EBV) infection is strongly linked to the development of nasopharyngeal carcinoma. While smoking and alcohol are known risk factors for many types of cancers, they are not particularly associated with nasopharyngeal carcinoma.

A 62-year-old male presents with increasing shortness of breath and a dry cough over the past year. He has a history of smoking and has no known exposure to asbestos or a history of collagen vascular diseases. Physical examination reveals bilateral inspiratory crackles in the lower lung fields. High-resolution computed tomography (HRCT) of the chest shows a pattern of usual interstitial pneumonia (UIP) with patchy, progressive bilateral interstitial fibrosis. A lung biopsy confirms the UIP pattern. What is the most likely diagnosis for this patient? A) Asbestosis B) Collagen vascular disease-associated interstitial lung disease C) Idiopathic pulmonary fibrosis (IPF) D) Nonspecific interstitial pneumonia E) Lymphocytic interstitial pneumonia

C) Idiopathic pulmonary fibrosis (IPF). The patient's presentation, history, radiologic findings, and histologic pattern of UIP suggest a diagnosis of IPF, which is a pulmonary disorder of unknown etiology characterized by patchy, progressive bilateral interstitial fibrosis. It is also known as cryptogenic fibrosing alveolitis and is a diagnosis of exclusion.

Which of the following is a common early event in the development of smoking-related carcinomas of the lung? A) Mutations in the TP53 tumor suppressor gene B) Mutations in the KRAS oncogene C) Inactivation of the putative tumor suppressor genes located on chromosome 3 (3p) D) Mutations activating the epidermal growth factor receptor (EGFR) E) Mutations activating other tyrosine kinases, including ALK, ROS1, HER2, or c-MET

C) Inactivation of the putative tumor suppressor genes located on chromosome 3 (3p). This is a common early event in the development of smoking-related carcinomas of the lung. Mutations in the TP53 tumor suppressor gene and the KRAS oncogene, on the other hand, occur relatively late. EGFR mutations are

What is the characteristic appearance of Pneumocystis jiroveci infection in lung tissue sections? A) Neutrophilic infiltrates B) Granulomatous inflammation C) Intraalveolar foamy, pink-staining exudate D) Caseating necrosis E) Giant cells

C) Intraalveolar foamy, pink-staining exudate. The involved areas of the lung in Pneumocystis jiroveci infection contain a characteristic intraalveolar foamy, pink-staining exudate with hematoxylin-eosin (H&E) stain ("cotton candy" exudate). The septa are thickened by edema and a minimal mononuclear infiltrate.

A 40-year-old female presents with painful, red, and tender nodules on the anterior aspect of her legs. Additionally, she reports dry eyes and dry mouth. On examination, bilateral parotid gland enlargement is noted. Further investigation reveals noncaseating granulomas in a skin biopsy and elevated serum calcium levels. What is the most likely diagnosis? A) Systemic lupus erythematosus B) Sjögren's syndrome C) Sarcoidosis D) Rheumatoid arthritis E) Lymphoma

C) Sarcoidosis. The patient's clinical presentation of erythema nodosum, ocular symptoms (sicca syndrome), parotid gland enlargement, and presence of noncaseating granulomas in a skin biopsy, along with elevated serum calcium levels, are consistent with sarcoidosis.

A 68-year-old man, a former shipyard worker, presents with progressive dyspnea and chest pain. He has a 30-year history of asbestos exposure. CT scan reveals extensive pleural fibrosis and plaque formation. Biopsy of the pleura shows spindled, occasionally fibroblastic-appearing cells growing in sheets. Based on the patient's history, imaging findings, and biopsy results, what is the most likely diagnosis? A) Pulmonary Adenocarcinoma B) Epithelial type Malignant Mesothelioma C) Sarcomatous type Malignant Mesothelioma D) Biphasic type Malignant Mesothelioma E) Pulmonary Fibrosis

C) Sarcomatous type Malignant Mesothelioma The patient's history of asbestos exposure, symptoms, and imaging findings suggest malignant mesothelioma. The biopsy showing spindled, occasionally fibroblastic-appearing cells growing in sheets is characteristic of the sarcomatous type of malignant mesothelioma. This type is less common than the epithelial type but should be considered given the biopsy findings.

A 52-year-old woman, who has worked in a ceramics factory for 25 years, presents with a worsening dry cough and increasing shortness of breath. She denies any history of smoking. On physical examination, fine inspiratory crackles are heard in both lung fields. A chest X-ray reveals small nodular opacities in the upper lung zones. A lung biopsy is performed, and microscopic examination shows concentrically arranged hyalinized collagen fibers with a whorled appearance surrounding an amorphous center. What is the most likely diagnosis? A) Asbestosis B) Coal Worker's Pneumoconiosis (CWP) C) Silicosis D) Tuberculosis E) Sarcoidosis

C) Silicosis. The patient's occupational history, clinical presentation, imaging findings, and lung biopsy results are consistent with silicosis. The distinctive "whorled" appearance of the collagen fibers surrounding an amorphous center is characteristic of silicotic nodules.

A 55-year-old man with a 30-pack year smoking history presents with persistent cough and weight loss. Imaging reveals a centrally located lung mass with mediastinal lymphadenopathy. Biopsy reveals small cells with scant cytoplasm, fine granular chromatin, and numerous mitotic figures. Despite aggressive chemotherapy, the patient's disease progresses rapidly, and he dies within a year of diagnosis. Based on the patient's history, symptoms, and biopsy findings, what is the most likely diagnosis? A) Squamous cell carcinoma of the lung B) Adenocarcinoma of the lung C) Small cell lung carcinoma (SCLC) D) Non-small cell lung carcinoma (NSCLC) E) Mesothelioma

C) Small cell lung carcinoma (SCLC). The patient's history of heavy smoking, presenting symptoms, and the biopsy findings (small cells with scant cytoplasm, fine granular chromatin, and numerous mitotic figures) are indicative of SCLC. Despite being very sensitive to chemotherapy, SCLCs typically have a poor prognosis, with a median survival of only 1 year.

A 62-year-old female with a 40-year history of smoking presents with a persistent cough, weight loss, and episodes of flushing. A chest CT scan reveals a centrally located mass in the right lung. A biopsy of the mass demonstrates small cells with scant cytoplasm and finely granular chromatin. Immunohistochemical staining shows positive neuroendocrine markers. Based on the clinical presentation and biopsy findings, what is the most likely diagnosis? A) Adenocarcinoma B) Squamous cell carcinoma C) Small cell lung carcinoma D) Large cell neuroendocrine carcinoma E) Metastatic carcinoma from another primary site

C) Small cell lung carcinoma. The patient's history of smoking, symptoms, location of the mass, and histological findings are characteristic of small cell lung carcinoma.

A 65-year-old male with a long history of smoking presents with a persistent cough and recent onset of weight loss. A chest X-ray reveals a centrally located mass in his right lung. Bronchoscopic biopsy reveals a well-differentiated squamous cell neoplasm with keratin pearls and intercellular bridges. Based on the clinical presentation and biopsy findings, what is the most likely diagnosis? A) Adenocarcinoma B) Small cell lung carcinoma C) Squamous cell carcinoma D) Large cell carcinoma E) Carcinoid tumor

C) Squamous cell carcinoma. The patient's smoking history, location of the mass, and histological findings are characteristic of squamous cell carcinoma of the lung.

Which of the following is NOT a typical characteristic of squamous cell carcinomas of the lung? A) More common in men than in women B) Closely correlated with a smoking history C) Tends to arise peripherally in minor bronchioles D) Often preceded by years of squamous metaplasia or dysplasia in the bronchial epithelium E) May show keratin pearls and intercellular bridges on histologic examination

C) Tends to arise peripherally in minor bronchioles. This is incorrect as squamous cell carcinomas of the lung tend to arise centrally in major bronchi.

What is the most important entity within the spectrum of chronic pneumonias? A) Pneumocystis pneumonia B) Aspergillus pneumonia C) Tuberculosis D) Histoplasmosis E) Nocardia pneumonia

C) Tuberculosis

What is the immunologic basis of hypersensitivity pneumonitis? A) IgE-mediated hypersensitivity reaction B) Type III hypersensitivity reaction C) Type IV hypersensitivity reaction D) Type II hypersensitivity reaction E) None of the above

C) Type IV hypersensitivity reaction. Several lines of evidence suggest that hypersensitivity pneumonitis is an immunologically mediated disease. Bronchoalveolar lavage specimens consistently show increased numbers of CD4+ and CD8+ T lymphocytes. Most affected patients have specific antibodies against the offending antigen in their serum. Complement and immunoglobulins have been demonstrated within vessel walls by immunofluorescence. Noncaseating granulomas are found in the lungs of two-thirds of affected patients. The immunologic basis of hypersensitivity pneumonitis is a type IV hypersensitivity reaction, which is mediated by T lymphocytes.

A 48-year-old male patient presents with persistent cough, hemoptysis, and shortness of breath. Chest CT reveals a mass in the main bronchus. A biopsy is taken and histological examination shows nests of uniform cells with regular round nuclei, "salt-and-pepper" chromatin, and rare mitoses. Based on the histological findings, what is the most likely diagnosis? A) Small cell lung carcinoma B) Atypical carcinoid tumor C) Typical carcinoid tumor D) Large cell neuroendocrine carcinoma E) Squamous cell carcinoma

C) Typical carcinoid tumor The histological findings of nests of uniform cells with regular round nuclei, "salt-and-pepper" chromatin, and rare mitoses are characteristic of a typical carcinoid tumor. These features differentiate it from the other options: small cell lung carcinoma, atypical carcinoid tumor, large cell neuroendocrine carcinoma, and squamous cell carcinoma.

OBSTRUCTIVE LUNG (AIRWAY) DISEASES chronic obstructive pulmonary disease (COPD) vs asthma

COPD = irreversible pulmonary damage asthma = reversible

OBSTRUCTIVE LUNG (AIRWAY) DISEASES: Emphysema: Conditions Related to Emphysema: • Mediastinal (interstitial) emphysema - Caused by - Etiology

Caused by entry of air into the interstitium of the lung, from where it may track to the mediastinum and sometimes the subcutaneous tissue. It may occur spontaneously if a sudden increase in Intraalveolar pressure (as with vomiting or violent coughing) produces alveolar rupture, which allows air to dissect into the interstitium. - Sometimes it develops in children with whooping cough - It may also occur in patients on respirators who have partial bronchiolar obstruction or in individuals with a perforating injury (e.g., a fractured rib). - When the interstitial air gets into the subcutaneous tissue, the patient may literally blow up like a balloon, with marked swelling of the head and neck and crackling crepitation over the chest (subcutaneous emphysema). (In most instances the air is resorbed spontaneously after the site of entry seals)

OBSTRUCTIVE LUNG (AIRWAY) DISEASES: Chronic Bronchitis Chronic bronchiolitis (small airway disease) Morphology:

Characterized by (1) goblet cell metaplasia (2) mucous plugging (3) inflammation (4) fibrosis - In severe cases, there may be complete obliteration of the lumen as a consequence of fibrosis (bronchiolitis obliterans).

OBSTRUCTIVE LUNG (AIRWAY) DISEASES Chronic Bronchitis vs Emphysema: The anatomic distribution of Chronic Bronchitis vs Emphysema Main cause of both

Chronic bronchitis initially involves the large airways Emphysema affects the acinus. - In severe or advanced cases of both, small airway disease (chronic bronchiolitis) is also present. - Although emphysema may exist without chronic bronchitis (particularly in inherited α1-anti-trypsin defciency) and vice versa, the two diseases usually coexist. - This is almost certainly because cigarette smoking is the major underlying cause of both

OBSTRUCTIVE LUNG (AIRWAY) DISEASES: Chronic Bronchitis Definition and Epidemiology

Chronic bronchitis is diagnosed on clinical grounds: it is defined by the presence of a persistent productive cough for at least 3 consecutive months in at least 2 consecutive years. It is common among cigarette smokers and urban dwellers in smog-ridden cities; some studies indicate that 20% to 25% of men in the 40- to 65-year-old age group have the disease.

RESTRICTIVE PULMONARY DISEASES: Chronic restrictive diseases:

Chronic restrictive diseases include the (1) pneumoconiosis (2) interstitial fibrosis of unknown etiology (3) infiltrative conditions such as sarcoidosis (formation of granuloma)

Pulmonary acini Components

Composed of respiratory bronchioles (arising from the terminal bronchiole) that proceed into alveolar ducts, which immediately branch into alveolar sacs, the blind ends of the respiratory passages, whose walls are formed entirely of alveoli, the ultimate site of gas exchange

A 65-year-old woman presents with a persistent cough and unexplained weight loss. A CT scan of the chest reveals a peripherally located, 2.8 cm mass in her left lung. A biopsy of the mass shows a well-demarcated focus of epithelial proliferation, composed of cuboidal to low-columnar cells, demonstrating nuclear hyperchromasia, pleomorphism, and prominent nucleoli. Genetic analyses reveal KRAS mutations. Based on the clinical presentation and biopsy findings, what is the most likely diagnosis? A) Small cell lung carcinoma B) Squamous cell carcinoma of the lung C) Atypical Adenomatous Hyperplasia (AAH) D) Adenocarcinoma In Situ (AIS) E) Invasive adenocarcinoma

D) Adenocarcinoma In Situ (AIS). The patient's clinical presentation, along with the histopathological findings and genetic analysis, are consistent with AIS. This form of adenocarcin

Which clinical manifestation of fungal infection is associated with Type I hypersensitivity against the fungus growing in the bronchi? A) Rhinocerebral mucormycosis B) Pulmonary mucormycosis C) Invasive aspergillosis D) Allergic bronchopulmonary aspergillosis E) Aspergilloma

D) Allergic bronchopulmonary aspergillosis. This condition occurs in patients with asthma who develop an exacerbation of symptoms caused by a Type I hypersensitivity against Aspergillus growing in the bronchi. Such patients often have circulating IgE antibodies against Aspergillus and peripheral eosinophilia.

Which of the following infections is more likely to occur in an HIV-infected individual with a CD4+ T cell count below 50 cells/μL? A) Bacterial pneumonia B) Tubercular infection C) Pneumocystis pneumonia D) Cytomegalovirus (CMV) infection E) Kaposi sarcoma

D) Cytomegalovirus (CMV) infection. In HIV-infected individuals, CMV and M. avium complex infections are uncommon until the very late stages of the disease, typically when the CD4+ T cell count drops below 50 cells/μL.

Which of the following cell types is NOT typically affected by cytomegalovirus (CMV) infection? A) Parenchymal epithelial cells in glandular organs B) Neurons in the brain C) Alveolar macrophages and epithelial and endothelial cells in the lungs D) Erythrocytes in the blood E) Tubular epithelial and glomerular endothelial cells in the kidneys

D) Erythrocytes in the blood. CMV typically infects various cell types depending on the organ, including parenchymal epithelial cells in glandular organs, neurons in the brain, alveolar macrophages and epithelial and endothelial cells in the lungs, and tubular epithelial and glomerular endothelial cells in the kidneys. Erythrocytes in the blood are not commonly affected by CMV infection.

Which factor does NOT influence the development of secondary tuberculosis? A) Primary tuberculosis B) Reactivation of dormant primary lesions C) Reinfection D) Genetic predisposition to lung cancer E) Exposure to a large inoculum of virulent bacilli

D) Genetic predisposition to lung cancer. Secondary tuberculosis may develop after primary tuberculosis, reactivation of dormant primary lesions, or reinfection. It is not influenced by a genetic predisposition to lung cancer.

A 4-year-old girl presents with a harsh, persistent cough and inspiratory stridor. She has been unwell with a cold for a few days. What is the most likely cause of her symptoms? A) Acute bacterial epiglottitis B) Tuberculous laryngitis C) Diphtheritic laryngitis D) Laryngotracheobronchitis (Croup) E) Streptococcal pharyngitis

D) Laryngotracheobronchitis (Croup) The child's age, symptoms, and recent upper respiratory tract infection suggest croup, which is most commonly caused by the parainfluenza virus.

What is the most frequent form of extrapulmonary tuberculosis? A) Systemic miliary tuberculosis B) Isolated-organ tuberculosis C) Pott disease D) Lymphadenitis E) Intestinal tuberculosis

D) Lymphadenitis. It is the most frequent form of extrapulmonary tuberculosis and typically occurs in the cervical region. Lymphadenopathy tends to be unifocal, and most patients do not have concurrent extranodal disease.

Which of the following pathogens is NOT commonly associated with hospital-acquired pneumonia? A) Enterobacteriaceae B) Pseudomonas spp. C) S. aureus D) S. pneumoniae E) All of the above are commonly associated

D) S. pneumoniae. In hospital-acquired pneumonias, S. pneumoniae is not a common pathogen. Gram-negative rods (Enterobacteriaceae and Pseudomonas spp.) and S. aureus are more common in hospital settings.

What is the characteristic histological feature of laryngeal papilloma? A) Smooth, hemispherical protrusions covered by intact mucosa B) Fibrous tissue with chronic irritation C) Raspberry-like excrescence with ulceration D) Slender, fingerlike projections supported by fibrovascular cores E) Multiple nodules caused by vertical transmission from an infected mother

D) Slender, fingerlike projections supported by fibrovascular cores Laryngeal papilloma histologically consists of these projections covered by typical stratified squamous epithelium. It forms a soft, raspberry-like excrescence on the true vocal cords, rarely exceeding 1 cm in diameter.

What is a typical characteristic of small cell lung carcinoma (SCLC)? A) They often present early with localized, resectable disease. B) They can be cured with lobectomy or pneumonectomy if detected early. C) They have a high rate of long-term remission with EGFR inhibitors. D) They have usually spread by the time of detection, making surgical resection unviable. E) They have a high survival rate with treatment, with most patients surviving over 10 years.

D) They have usually spread by the time of detection, making surgical resection unviable. SCLCs are typically advanced by the time they are detected. Surgical resection is usually not an option due to their extensive spread. They are sensitive to chemotherapy but invariably recur, and targeted therapies are currently unavailable for SCLC.

What is the target of circulating autoantibodies in Goodpasture Syndrome? A) Type I collagen B) Type II collagen C) Type III collagen D) Type IV collagen E) Type V collagen

D) Type IV collagen. In Goodpasture Syndrome, circulating autoantibodies target specific domains of type IV collagen, which are intrinsic to the basement membranes of renal glomeruli and pulmonary alveoli. This leads to the destruction and inflammation of the basement membranes, causing necrotizing hemorrhagic interstitial pneumonitis and rapidly progressive glomerulonephritis.

What is the radiologic and histologic pattern of fibrosis required for the diagnosis of idiopathic pulmonary fibrosis (IPF)? A) Nonspecific interstitial pneumonia B) Lymphocytic interstitial pneumonia C) Desquamative interstitial pneumonia D) Usual interstitial pneumonia E) Cryptogenic organizing pneumonia

D) Usual interstitial pneumonia (UIP). The radiologic and histologic pattern of fibrosis required for the diagnosis of IPF is UIP. It is important to note that similar pathologic changes may be present in other conditions, making IPF a diagnosis of exclusion.

Define Honeycomb Fibrosis

Dense fibrosis causes collapse of alveolar walls and formation of cystic spaces lined by hyperplastic type II pneumocytes or bronchiolar epithelium (honeycomb fibrosis).

Respiratory system: Development and Anatomy

Developmentally, the respiratory system is an outgrowth from the ventral wall of the foregut. The midline trachea develops two lateral outpouchings, the lung buds. The right lung bud divides into three main bronchi, and the left into two main bronchi. The main bronchi branch- bronchioles (which are distinguished from bronchi by the lack of cartilage and submucosal glands within their walls)- terminal bronchioles- the part of the lung distal to the terminal bronchiole is called an acinus.

OBSTRUCTIVE VERSUS RESTRICTIVE PULMONARY DISEASES: Classification

Diffuse pulmonary diseases can be classified into two categories: (1) obstructive (airway) disease, characterized by an increase in resistance to air flow caused by partial or complete obstruction at any level; (2) restrictive disease, characterized by reduced expansion of lung parenchyma and decreased total lung capacity.

ACUTE RESPIRATORY DISTRESS SYNDROME (ARDS) Difference between ARDS and respiratory distress syndrome of the newborn;

Distress syndrome of the newborn caused by a defciency of surfactant caused by prematurity.

OBSTRUCTIVE LUNG (AIRWAY) DISEASES: Asthma : Non-Atopic Asthma - Allergen sensitization - Skin test - Factors that trigger

Do not have evidence of allergen sensitization,. Skin test results usually are negative. - Respiratory infections due to viruses (e.g., rhinovirus, parainfluenza virus) and inhaled air pollutants (e.g., sulfur dioxide, ozone, nitrogen dioxide) are common triggers. Although the connections are not well understood, the ultimate humoral and cellular mediators of airway obstruction (e.g., eosinophils) are common to both atopic and nonatopic variants of asthma, so they are treated in a similar way.

OBSTRUCTIVE LUNG (AIRWAY) DISEASES: Asthma : Atopic form Pathogenesis The Early phase reaction (3)

Dominated by (1) bronchoconstriction (2) increased mucus production (3) vasodilation. Bronchoconstriction is triggered by mediators released from mast cells, including histamine, prostaglandin D2, and leukotrienes LTC4, D4, and E4, and also by reflex neural pathways.

OBSTRUCTIVE LUNG (AIRWAY) DISEASES: Emphysema: The classic presentation of emphysema with "bronchitis"

Dyspnea usually is less prominent, and in the absence of increased respiratory drive the patient retains carbon dioxide, becoming hypoxic and often cyanotic For unclear reasons, such patients tend to be obese—hence the designation "blue bloaters."

A 35-year-old female presents to the clinic with a 3-month history of low-grade fever, night sweats, malaise, anorexia, and weight loss. She also reports a productive cough with initially mucoid sputum, which has now become purulent. A chest X-ray reveals consolidation and cavitation in the apices of the lungs. Sputum samples are collected for further investigation. What is the most common diagnostic method for tuberculosis? A) PCR amplification B) Conventional cultures for mycobacteria C) Liquid media-based radiometric assays D) Fluorescent auramine rhodamine staining E) Acid-fast staining of sputum

E) Acid-fast staining of sputum. The most common diagnostic method for tuberculosis is the demonstration of acid-fast organisms in sputum by staining or by the use of fluorescent auramine rhodamine. Although other methods, such as PCR amplification and liquid media-based radiometric assays, can be used, acid-fast staining remains the standard diagnostic modality.

Which of the following statements about carcinoid tumors is accurate? A) They are composed of cells lacking neurosecretory granules. B) They are high-grade neuroendocrine carcinomas. C) Carcinoid tumors are typically not resectable and incurable. D) Bronchial carcinoids most commonly occur in elderly individuals. E) Carcinoid tumors are low-grade neuroendocrine carcinomas

E) Carcinoid tumors are low-grade neuroendocrine carcinomas and can occasionally occur as part of the multiple endocrine neoplasia syndrome. Carcinoid tumors are composed of cells containing dense-core neurosecretory granules, they are considered low-grade neuroendocrine carcinomas (not high-grade), and they are often resectable and curable. Bronchial carcinoids typically occur in young adults (average age 40 years), not elderly individuals.

Which of the following symptoms is not typically associated with carcinoid tumors? A) Cough B) Hemoptysis C) Recurrent bronchial and pulmonary infections D) Intermittent attacks of diarrhea, flushing, and cyanosis E) Chest pain

E) Chest pain The typical symptoms of carcinoid tumors are cough, hemoptysis, and recurrent bronchial and pulmonary infections due to their intraluminal growth. Occasionally, pulmonary carcinoids can induce the carcinoid syndrome, which is characterized by intermittent attacks of diarrhea, flushing, and cyanosis. Chest pain is not a common symptom of carcinoid tumors.

Which of the following risk factors does NOT predispose to venous thrombosis in the legs and subsequently to pulmonary embolism? A) Prolonged bed rest with immobilization of the legs B) Surgery, especially orthopedic surgery on the knee or hip C) Severe trauma, including burns or multiple fractures D) Congestive heart failure E) Hypertension

E) Hypertension. The risk factors listed in the text that predispose to venous thrombosis in the legs and subsequently to pulmonary embolism include prolonged bed rest with immobilization of the legs, surgery (especially orthopedic surgery on the knee or hip), severe trauma (including burns or multiple fractures), congestive heart failure, the period around parturition or use of oral contraception pills with high estrogen content in women, disseminated cancer, and primary disorders of hypercoagulability (e.g., factor V Leiden).

Which of the following is a self-limited upper respiratory tract infection caused by Legionella pneumophila? A) Legionnaire disease B) Pseudomonas pneumonia C) Klebsiella-related pneumonia D) Staphylococcal pneumonia E) Pontiac fever

E) Pontiac fever. Pontiac fever is a self-limited upper respiratory tract infection caused by Legionella pneumophila. It does not cause pneumonic symptoms and is characterized by fever, headache, and myalgias.

Which of the following statements correctly describes the histological features of typical carcinoid tumors? A) They show a high mitotic rate and small foci of necrosis. B) They display substantial pleomorphism and frequent mitoses. C) They contain TP53 mutations in a significant proportion of cases. D) They have irregular nuclei with "salt-and-pepper" chromatin. E) They are composed of nests of uniform cells with regular round nuclei with "salt-and-pepper" chromatin, and show rare mitoses and little pleomorphism.

E) They are composed of nests of uniform cells with regular round nuclei with "salt-and-pepper" chromatin, and show rare mitoses and little pleomorphism. Typical carcinoids, unlike atypical carcinoids, are composed of nests of uniform cells that have regular round nuclei with "salt-and-pepper" chromatin, absent or rare mitoses, and little pleomorphism. They do not show a high mitotic rate, substantial pleomorphism, or contain TP53 mutations, which are characteristics of atypical carcinoids.

OBSTRUCTIVE LUNG (AIRWAY) DISEASES: Chronic Bronchitis Pathogenesis

Environmental irritants (Air pollutants\Tobacco particles) induce hypertrophy of mucous glands in the trachea and bronchi as well as an increase in mucin-secreting goblet cells in the epithelial surfaces of smaller bronchi and bronchioles. - These irritants also cause inflammation marked by the infiltration of macrophages, neutrophils, and lymphocytes.

OBSTRUCTIVE LUNG (AIRWAY) DISEASES: Emphysema: 4 Factors that influence the development of emphysema:

Factors that influence the development of emphysema include the following: • Inflammatory cells and mediators The inflammatory cells present in lesions include neutrophils, macrophages, and CD4+ and CD8+ T cells • Protease-anti-protease imbalance Several proteases are released from the infammatory cells and epithelial cells that break down connective tissues. In patients who develop emphysema, there is a relative defciency of protective anti-proteases • Oxidative stress Reactive oxygen species are generated by cigarette smoke and other inhaled particles and released from activated inflammatory cells such as macrophages and neutrophils. These cause additional tissue damage and inflammation • Airway infection Although infection is not thought to play a role in the initiation of tissue destruction, bacterial and/or viral infections cause acute exacerbations.

ACUTE RESPIRATORY DISTRESS SYNDROME (ARDS) Pathogenesis Early histologic signs

Histologic examination of lungs early in the disease process shows Increased numbers of neutrophils within the vascular space, the interstitium, and the alveoli.

OBSTRUCTIVE LUNG (AIRWAY) DISEASES: Emphysema: Complication - Death reasons

Hypoxia-induced pulmonary vascular spasm and loss of pulmonary capillary surface area from alveolar destruction causes the gradual development of secondary pulmonary hypertension, which in 20% to 30% of patients leads to right-sided congestive heart failure (cor pulmonale) - Death from emphysema is related to either respiratory failure or right-sided heart failure.

ACUTE RESPIRATORY DISTRESS SYNDROME (ARDS) Pathogenesis

In ARDS, the integrity (שלמות) of the alveolar-capillary membrane is compromised by endothelial and epithelial injury. - Most work suggests that ARDS stems from an inflammatory reaction initiated by a variety of pro-inflammatory mediators - Neutrophils are thought to have an important role in the pathogenesis of ARDS. 1. 30 minutes after an acute insult, there is increased synthesis of interleukin 8 (IL-8), a potent neutrophil chemotactic and activating agent, by pulmonary macrophages. 2. Release of IL-8 and other factors, such as IL-1 and tumor necrosis factor (TNF), leads to endothelial activation and sequestration and activation of neutrophils in pulmonary capillaries. 3. Activated neutrophils release a variety of products (e.g., reactive oxygen species, proteases) that damage the alveolar epithelium and endothelium. 4. The assault (ההתקפה) on the endothelium and epithelium causes vascular leakiness and loss of surfactant that results in the inability of alveolar unit to expand.

OBSTRUCTIVE LUNG (AIRWAY) DISEASES: Emphysema: (2) Panacinar (panlobular) emphysema - Main location of damage - Associated with ? deficiency

In contrast to centriacinar emphysema, panacinar emphysema occurs more commonly in the lower lung zones and is associated with α1-anti-trypsin defciency.

OBSTRUCTIVE LUNG (AIRWAY) DISEASES: Chronic Bronchitis Different from asthma by

In contrast with asthma, eosinophils are not seen in chronic bronchitis

OBSTRUCTIVE LUNG (AIRWAY) DISEASES: Chronic Bronchitis Early signs

In early stages of the disease, the cough raises mucoid sputum, but airflow is not obstructed. Some patients with chronic bronchitis have evidence of hyperresponsivity airways, with intermittent (לסירוגין) bronchospasm and wheezing (asthmatic bronchitis), while other bronchitis patients, especially heavy smokers, develop chronic outflow obstruction, usually with associated emphysema (COPD)

OBSTRUCTIVE LUNG (AIRWAY) DISEASES: Chronic Bronchitis Bronchiolitis obliterans

In severe Chronic bronchiolitis cases, there may be complete obliteration of the lumen as a consequence of fibrosis (bronchiolitis obliterans). It is the submucosal fibrosis that leads to luminal narrowing and airway obstruction. Emphysematous changes often coexist.

Lobar Pneumonia vs Lobular Bronchopneumonia: (2)

In the context of pneumonias, the term "consolidation," used frequently, refers to "solidification" of the lung due to replacement of the air by exudate in the alveoli. 1. Patchy consolidation of the lung is the dominant characteristic of bronchopneumonia, 2. Consolidation of a large portion of a lobe or of an entire lobe defines lobar pneumonia

ACUTE RESPIRATORY DISTRESS SYNDROME (ARDS) Morphology: Organizing stage and Resolution

In the organizing stage, type II pneumocytes proliferate vigorously in an attempt to regenerate the alveolar lining. Resolution is unusual; - More commonly, the fibrin-rich exudates organize into Intraalveolar fibrosis. Marked thickening of the alveolar septa resulted due to proliferation of interstitial cells and deposition of collagen.

OBSTRUCTIVE LUNG (AIRWAY) DISEASES chronic obstructive pulmonary disease (COPD) Definition

In view of their propensity to coexist, emphysema and chronic bronchitis often are grouped together under the rubric of chronic obstructive pulmonary disease (COPD).

OBSTRUCTIVE LUNG (AIRWAY) DISEASES: Emphysema: (4) Irregular emphysema - Main features

Irregular emphysema, so named because the acinus is irregularly involved, is almost invariably associated with scarring Although clinically asymptomatic, this may be the most common form of emphysema

OBSTRUCTIVE LUNG (AIRWAY) DISEASES: Emphysema: 4 Types

It is classified according to its anatomic distribution. (1) centriacinar (most common w\ smoking) (2) panacinar (3) distal acinar (4) irregular (most common form of emphysema) - Only Centriacinar and Panacinar cause signifcant airway obstruction, with centriacinar emphysema being about 20 times more common than panacinar disease.

ACUTE RESPIRATORY DISTRESS SYNDROME (ARDS) Garde of disease based on

It is graded based on the severity of the changes in arterial blood oxygenation.

OBSTRUCTIVE LUNG (AIRWAY) DISEASES: Chronic Bronchitis Morphology:

Larger airways: - The mucosal lining of the larger airways usually is hyperemic and swollen by edema fluid and is covered by a layer of mucinous or mucopurulent secretions. - The diagnostic feature of chronic bronchitis in the trachea and larger bronchi is enlargement of the mucus-secreting glands (Fig). Smaller Bronchi and Bronchioles: The smaller bronchi and bronchioles also may be filled with secretions Lymphocytes and macrophages and admixed neutrophils infiltration are frequently seen in the bronchial mucosa.

OBSTRUCTIVE LUNG (AIRWAY) DISEASES: Asthma Major factors contributing to the development of asthma

Major factors contributing to the development of asthma include (1) genetic predisposition to type I hypersensitivity (atopy) (2) acute and chronic airway inflammation (3) bronchial hyperresponsiveness to a variety of stimuli.

ACUTE RESPIRATORY DISTRESS SYNDROME (ARDS) Survival consequences

Most patients who survive the acute insult recover normal respiratory function within 6 to 12 months, but the rest develop diffuse interstitial fibrosis leading to chronic respiratory insufficiency.

OBSTRUCTIVE LUNG (AIRWAY) DISEASES: Emphysema: The classic presentation of emphysema with no "bronchitis" component

Patient is barrel-chested and dyspneic, with prolonged expiration, sitting forward in a hunched-over position (עמדה שפופה). - In these patients, air space enlargement is severe and diffusing capacity is low. - Dyspnea and hyperventilation are prominent, so that until very late in the disease, gas exchange is adequate and blood gas values are relatively normal - Because of prominent dyspnea and adequate oxygenation of hemoglobin, these patients sometimes are sometimes called "pink puffers."

OBSTRUCTIVE LUNG (AIRWAY) DISEASES: Emphysema: Characteristics:

Permanent enlargement of the air spaces distal to the terminal bronchioles, accompanied by destruction of their walls without significant fibrosis.

OBSTRUCTIVE LUNG (AIRWAY) DISEASES: Emphysema: Protease-mediated damage of extracellular matrix

Protease-mediated damage of extracellular matrix has a central role in the airway obstruction seen in emphysema. Small airways are normally held open by the elastic recoil of the lung parenchyma, and the loss of elastic tissue in the walls of alveoli that surround respiratory bronchioles reduces radial traction and thus causes the respiratory bronchioles to collapse during expiration. This leads to functional airflow obstruction despite the absence of mechanical obstruction.

ACUTE RESPIRATORY DISTRESS SYNDROME (ARDS) Definition

Respiratory failure occurring within 1 week of a known clinical insult with bilateral opacities (אטימות דו-צדדית) on chest imaging, not fully explained by effusions, atelectasis, cardiac failure, or fluid overload. Causes are diverse; the shared feature is that all lead to extensive bilateral injury to alveoli.

RESTRICTIVE PULMONARY DISEASES: - Main Etiologies

Restrictive defects occur in two general conditions: (1) chest wall disorders in the presence of normal lungs (e.g., with severe obesity, diseases of the pleura, and neuromuscular disorders, such as the Guillain-Barre syndrome [Chapter 22], that affect the respiratory muscles) - "PAINT" mnemonic (2) acute or chronic interstitial lung diseases. The classic acute restrictive disease is ARDS.

ACUTE RESPIRATORY DISTRESS SYNDROME (ARDS) Severe ARDS characteristics

Severe ARDS is characterized by : (1) rapid onset of life-threatening respiratory insufficiency (2) cyanosis (3) severe arterial hypoxemia that is refractory to oxygen therapy (שאינה עמידה לטיפול בחמצן.).

OBSTRUCTIVE LUNG (AIRWAY) DISEASES: Chronic Bronchitis Pathogenesis: the airflow obstruction in chronic bronchitis results from

The airflow obstruction in chronic bronchitis results from (1) small airway disease (chronic bronchiolitis) induced by mucous plugging of the bronchiolar lumen, inflammation, and bronchiolar wall fibrosis (2) coexistent emphysema In general, while small airway disease (chronic bronchiolitis) is an important component of early, mild airflow obstruction, chronic bronchitis with significant airflow obstruction almost always is complicated by emphysema.

OBSTRUCTIVE LUNG (AIRWAY) DISEASES: Emphysema: (3) Distal acinar (paraseptal) emphysema autopsy

The chest cavity is opened at autopsy to reveal numerous large bullae apparent on the surface of the lungs in a patient dying with Distal acinar (paraseptal) emphysema. Bullae are large dilated airspaces that bulge out from beneath the pleura. Emphysema is characterized by a loss of lung parenchyma by destruction of alveoli so that there is permanent dilation of airspaces with loss of elastic recoil.

OBSTRUCTIVE LUNG (AIRWAY) DISEASES: Asthma : Atopic form Pathogenesis

The classic, Atopic form is associated with excessive type 2 helper T (TH2) cell activation results in (1) IL-4 and IL-13 stimulates IgE production (2) IL-5 activates eosinophils (3) IL-13 stimulates mucus production Mast cell-derived mediators produce two waves of reaction: an early (immediate) phase and a late phase.

OBSTRUCTIVE LUNG (AIRWAY) DISEASES: Bronchiectasis - The conditions that most commonly predispose to bronchiectasis include:

The conditions that most commonly predispose to bronchiectasis include: • Bronchial obstruction. Common causes are tumors, foreign bodies, and impaction of mucus. In these conditions, bronchiectasis is localized to the obstructed lung segment. Bronchiectasis also may complicate atopic asthma and chronic bronchitis. • Congenital or hereditary conditions —for example: a. Cystic fibrosis, in which widespread severe bronchiectasis results from obstruction caused by abnormally viscid mucus and secondary infections b. Immunodeficiency states, particularly immunoglobulin deficiencies, in which localized or diffuse bronchiectasis often develops because of recurrent bacterial infections c. Primary ciliary dyskinesia (also called the immotile cilia syndrome). This is a rare autosomal recessive disorder that is frequently associated with bronchiectasis and with sterility in males. It is caused by inherited abnormalities of cilia that impair mucociliary clearance of the airways, leading to persistent infections. • Necrotizing, or suppurative, pneumonia, particularly with virulent organisms such as Staphylococcus aureus or Klebsiella spp., predispose affected patients to development of bronchiectasis. Posttuberculosis bronchiectasis continues to be a significant cause of morbidity in endemic areas.

ACUTE RESPIRATORY DISTRESS SYNDROME (ARDS) Pathogenesis - The destructive forces unleashed by neutrophils can be counteracted by

The destructive forces unleashed (ששוחררו) by neutrophils can be counteracted by an array of endogenous anti-proteases and anti-oxidants that are upregulated by proinflammatory cytokines. In the end, it is the balance between the destructive and protective factors that determines the degree of tissue injury and clinical severity of the ARDS.

OBSTRUCTIVE LUNG (AIRWAY) DISEASES: Emphysema: (3) Distal acinar (paraseptal) emphysema - Main location of damage - Characteristic finding

The emphysema is more striking adjacent to the pleura, along the lobular connective tissue septa, and at the margins of the lobules. - It occurs adjacent to areas of fibrosis, scarring, or atelectasis and is usually more severe in the upper half of the lungs. - The characteristic finding is the presence of multiple, contiguous, enlarged air spaces ranging in diameter from less than 0.5 mm to more than 2.0 cm, sometimes forming cystic structures that, with progressive enlargement, give rise to bullae

OBSTRUCTIVE LUNG (AIRWAY) DISEASES: Emphysema: - Genetic defciency of the anti-protease α1-anti-trypsin - α1-anti-trypsin Functions

The idea that proteases are important is based in part on the observation that patients with a genetic defciency of the anti-protease α1-anti-trypsin have a predisposition to develop pulmonary emphysema, which is compounded by smoking. - About 1% of all patients with emphysema have this defect. - α1-anti-trypsin, normally present in serum, tissue fluids, and macrophages, is a major inhibitor of proteases (particularly elastase) secreted by neutrophils during inflammation.

ACUTE RESPIRATORY DISTRESS SYNDROME (ARDS) Morphology: Acute phase of ARDS Hyaline Membrane

The most characteristic finding is the presence of hyaline membranes, particularly lining the distended alveolar ducts Such membranes consist of fibrin-rich edema fluid admixed with remnants of necrotic epithelial cells (the picture is remarkably similar to that seen in respiratory distress syndrome of the newborn)

ACUTE RESPIRATORY DISTRESS SYNDROME (ARDS) The most frequent triggers of ARDS

The most frequent triggers of ARDS are (1) pneumonia (35%-45%) (2) sepsis (30%-35%), (3) aspiration (4) trauma (including brain injury, abdominal surgery, and multiple fractures) (5) pancreatitis (6) transfusion reactions.

OBSTRUCTIVE LUNG (AIRWAY) DISEASES: Asthma : Atopic form - Definition - Onset of asthmatic attacks is often preceded by - Attacks may be triggered by - Atopic asthma also can be diagnosed based on - A skin test

This is the most common type of asthma and is a classic example of type I IgE-mediated hypersensitivity reaction. - It usually begins in childhood - Onset of asthmatic attacks is often preceded by allergic rhinitis (נזלת), urticaria, or eczema - Attacks may be triggered by allergens in dust, pollen, animal dander, or food, or by infections. - Atopic asthma also can be diagnosed based on serum radioallergosorbent tests (RASTs) that identify the presence of IgEs that recognize specific allergens. - A skin test with the offending antigen results in an immediate wheal-and-fare reaction

OBSTRUCTIVE LUNG (AIRWAY) DISEASES: Bronchiectasis Pathogenesis

Two intertwined processes contribute to bronchiectasis: obstruction and chronic infection. For example, obstruction caused by a foreign body impairs clearance of secretions, providing a favorable substrate for superimposed infection. The resultant inflammatory damage to the bronchial wall and the accumulating exudate further distend the airways, leading to irreversible dilation. Conversely, a persistent necrotizing infection in the bronchi or bronchioles may lead to poor clearance of secretions, obstruction, and inflammation with peribronchial fibrosis and traction on the bronchi, culminating again in full-blown bronchiectasis.

OBSTRUCTIVE LUNG (AIRWAY) DISEASES: Emphysema: Centriacinar Morphology

Until late stages, the lungs are a deeper pink than in panacinar emphysema and less voluminous, and the upper two-thirds of the lungs are more severely affected than the lower lungs - Destruction of alveolar walls without fibrosis, leading to enlarged air spaces (picture) - Terminal and respiratory bronchioles may be deformed because of the loss of septa that help tether (לִקְשׁוֹר) these structures in the parenchyma. - Bronchiolar inflammation and submucosal fibrosis are consistently present in advanced disease.

OBSTRUCTIVE LUNG (AIRWAY) DISEASES: Bronchiectasis Morphology Healing

When healing occurs, the lining epithelium may regenerate completely; however, the injury usually cannot be repaired and abnormal dilation and scarring persist. Fibrosis of the bronchial and bronchiolar walls and peribronchiolar fibrosis develop in more chronic cases In some instances the necrosis destroys the bronchial or bronchiolar walls, producing an abscess cavity

OBSTRUCTIVE LUNG (AIRWAY) DISEASES: Emphysema: Centriacinar Progression

With the loss of elastic tissue in the surrounding alveolar septa, radial traction on the small airways is reduced. As a result, they tend to collapse during expiration — an important cause of chronic airflow obstruction in severe emphysema - Bronchiolar inflammation and submucosal fibrosis are consistently present in advanced disease.

OBSTRUCTIVE LUNG (AIRWAY) DISEASES: Asthma Bronchial thermoplasty

a procedure that decreases the amount of constriction of the airways during an asthma attack by using thermal energy to reduce the muscle thickness in the bronchioles involves controlled delivery of thermal energy during bronchoscopy to reduce the mass of smooth muscle and airway responsiveness, is being evaluated in patients with severe, poorly controlled asthma.

ATELECTASIS (COLLAPSE) • Resorption atelectasis: - Etiology

a. (most frequently) Occurs postoperatively due to intrabronchial mucous or mucopurulent plugs b. Foreign body aspiration (particularly in children) c. Bronchial asthma d. Bronchiectasis e. Chronic bronchitis f. Intrabronchial tumor (in which it may be the first sign of malignancy).

RESTRICTIVE PULMONARY DISEASES: - Main characteristics of diffuse restrictive disorders

a. FVC is reduced b. Expiratory flow rate is normal or reduced proportionately. Hence, the ratio of FEV to FVC is near normal.

OBSTRUCTIVE PULMONARY DISEASES: - Main characteristics of diffuse obstructive disorders

a. Forced vital capacity (FVC) is either normal or slightly decreased b. Expiratory flow rate is significantly decreased. (usually measured as the forced expiratory volume at 1 second (FEV1), Thus, the ratio of FEV to FVC is characteristically decreased.

OBSTRUCTIVE LUNG (AIRWAY) DISEASES: Emphysema: (4) Irregular emphysema - Example

healed inflammatory diseases.

OBSTRUCTIVE LUNG (AIRWAY) DISEASES: Emphysema: (2) Panacinar (panlobular) emphysema - Main features

the acini are uniformly enlarged, from the level of the respiratory bronchiole to the terminal blind alveoli

OBSTRUCTIVE LUNG (AIRWAY) DISEASES: Emphysema: (3) Distal acinar (paraseptal) emphysema - Main features - most often in

the proximal portion of the acinus is normal but the distal part is primarily involved. most often in young adults who present with spontaneous pneumothorax.

Cytomegalovirus (CMV) Infection: Transmission (5)

• A fetus can be infected transplacentally from a newly acquired or reactivated infection in the mother (congenital CMV infection). • The virus can be transmitted to the infant through cervical or vaginal secretions at birth, or later through the breast milk of a mother with an active infection (perinatal CMV infection). • Preschool children, especially in day care centers, can acquire it through saliva. Toddlers so infected readily transmit the virus to their parents. • In patients older than 15 years of age, the venereal route is the dominant mode of transmission, but spread also may occur through contact with respiratory secretions and by the fecal-oral route. • Iatrogenic transmission can occur at any age through organ transplantation or blood transfusion.

OBSTRUCTIVE LUNG (AIRWAY) DISEASES: Emphysema: Conditions Related to Emphysema: • Bullous emphysema

• Bullous emphysema refers to any form of emphysema that produces large subpleural blebs or bullae (space>1 cm) Such blebs represent localized accentuations of one of the four forms of emphysema; most often the blebs are subpleural, and on occasion they may rupture, leading to pneumothorax.

OBSTRUCTIVE LUNG (AIRWAY) DISEASES: Emphysema: Conditions Related to Emphysema: • Compensatory emphysema

• Compensatory emphysema - Dilation of alveoli in response to loss of lung substance elsewhere, such as occurs after surgical removal of a diseased lung or lobe.

OBSTRUCTIVE LUNG (AIRWAY) DISEASES: Emphysema: Conditions Related to Emphysema: • Obstructive overinflation

• Obstructive overinflation - Expansion of the lung due to air trapping. A common cause is subtotal obstruction of an airway by a tumor or foreign object. Obstructive overinflation can be life-threatening if expansion of the affected portion produces compression of the remaining normal lung. results from partial obstruction of a bronchus or bronchiole, when it becomes more difficult for air to leave the alveoli than to enter; there is a gradual accumulation of air distal to the obstruction, the so-called bypass, ball valve, or check valve type of obstruction.

OBSTRUCTIVE LUNG (AIRWAY) DISEASES: Asthma : Atopic form Consequences: airway remodeling.

• Repeated bouts of inflammation lead to structural changes in the bronchial wall that are collectively referred to as airway remodeling: (1) hypertrophy of bronchial smooth muscle and mucus glands (2) increased vascularity and deposition of subepithelial collagen, which may occur as early as several years before initiation of symptoms.

ATELECTASIS (COLLAPSE) Classification

• Resorption atelectasis • Compression atelectasis • Contraction atelectasis

OBSTRUCTIVE LUNG (AIRWAY) DISEASES: Asthma : Atopic form Pathogenesis The Late phase reaction

• The late-phase reaction is inflammatory in nature. Inflammatory mediators stimulate epithelial cells to produce chemokines (including eotaxin, a potent chemoattractant and activator of eosinophils) that promote the recruitment of TH2 cells, eosinophils, and other leukocytes, thus amplifying an inflammatory reaction that is initiated by resident immune cells.

SUMMARY CARCINOMA OF THE LUNG

• The three major histologic subtypes are adenocarcinoma (most common), squamous cell carcinoma, and small cell carcinoma, each of which is clinically and genetically distinct. Adenocarcinomas are the most common cancers overall and are especially common in women and in nonsmokers. • Smoking is the most important risk factor for lung cancer. • Precursor lesions include atypical adenomatous hyperplasia and adenocarcinoma in situ for adenocarcinomas and squamous dysplasia for squamous cancer. • Tumors 3 cm or less in diameter characterized by pure growth along preexisting structures without stromal invasion are called adenocarcinoma in situ. • SCLCs are best treated with chemotherapy, because almost all are metastatic at presentation. The other carcinomas may be curable by surgery if limited to the lung. Targeted therapies, such as EGFR inhibitor therapy for adenocarcinomas with EGFR mutations, can be effective, an excellent example of personalized cancer therapy. Immunotherapies are under development and show promise. • Lung cancers commonly cause a variety of paraneoplastic syndromes.

OBSTRUCTIVE LUNG (AIRWAY) DISEASES: Asthma Airway remodeling includes:

• Thickening of airway wall • Sub-basement membrane fibrosis (Fig. 13.11) • Increased submucosal vascularity • An increase in size of the submucosal glands and goblet cell metaplasia of the airway epithelium • Hypertrophy and/or hyperplasia of the bronchial muscle


Ensembles d'études connexes

Saunders | Ethical & Legal Issues

View Set

PC Pro CHapter 3 (3.8.3.14) Questions

View Set

Lifespan Development: Chapter One

View Set

Ch10 - Collecting Data by Observation

View Set

BIOL&160 - SmartBook Assignment Chapter 11. DNA Technology

View Set